physiology

Ace your homework & exams now with Quizwiz!

26. A 20-year-old competitive cyclist taking recombinant human erythropoietin has polycythemia. Which of the following is the most likely cause? A) Early release of reticulocytes from bone marrow B) Increased erythrocyte life span C) Proliferation of bone marrow stem cells D) Proliferation of erythroid precursors

D) Proliferation of erythroid precursors

A 15-year-old boy is brought to the physician because of painless swelling in his left breast for 1 month. There is no family history of breast cancer, He is at the 65th percentile for height and 70th percentile for weight, His pulse is 65/min, and blood pressure is 118/76 mm Hg. Physical examination shows mild facial acne, thickening of mustache hair on the face, and the presence of axillary hair. There is no adenopathy. The left breast appears mildly enlarged with a palpable breast bud; the right breast is normal with no gynecomastia, His genitalia are Tanner stage 3, Which of the following is the most appropriate initial statement by the physician regarding this patient's breast finding?

A) "Have you tried smoking marijuana recently? That has been known to cause breast enlargement," B) "Some teenagers experiment with their parents' prescription drugs, even birth control pills. Are you?" C) "This is a common condition for teenage boys and should resolve in time." D) "This may be a sign of a chromosomal problem. We need to do some genetic testing." E) "This may be a sign of a more serious condition. Well have to do more tests to be sure." C) "This is a common condition for teenage boys and should resolve in time."

A 71-year-old woman comes to the physician for a follow-up examination, She has a 30-year history of type 2 diabetes mellitus currently treated with insulin. She is 160 cm (5 ft 3 in) tall and weighs 59 kg (130 lb); BMI is 23 kg/m2. Her blood pressure is 116/78 mm Hg. This patient is most likely to have an endogenous fasting serum insulin concentration (N=5-20 µU/mL) closest to which of the following? A) 1 µU/mL B) 10 µU/mL C) 20 µU/mL D) 40 µU/mL E) 80 µU/mL

A) 1 µU/mL

A 39-year-old man with polycystic kidney disease is brought to the physician for a follow-up examination, He has a 6-month history of intermittent blood in his urine. His temperature is 37°C (98.6T), pulse is 100/min, respirations are 24/min, and blood pressure is 160/90 mm Hg. Physical examination shows no other abnormalities. His serum urea nitrogen concentration is 100 mg/dL, and serum creatinine concentration is 8 mg/dL. Urinalysis shows blood, Arterial blood gas analysis on roam air is most likely to show which of the following sets of findings? PH Pco2(mm Hg) HCO3-(mEq/L) A) 7.22 28 11 B) 7.32 64 32 C) 7.38 40 23 D) 7.46 19 13 E) 7.49 50 37

A) 7.22 28 11

A 35-year-old man comes to the physician because of severe midline back pain during the past 4 weeks. He has gained 8 kg (18 lb) during the past 6 months. Visual field testing shows decreased peripheral vision in both eyes. An x-ray of the chest shows compression fractures at T10 and L1. There is a healing left rib fracture posteriorly at T6. Imaging studies show a pituitary macroadenoma. The most likely cause of his condition is a tumor producing which of the following hormones? A) ACTH B) Growth hormone C) Luteinizing hormone D) Prolactin E) Thyroid-stimulating hormone

A) ACTH

A 51-year-old man comes to the physician because of a 2-week history of cough productive of blood. He also has had a 3-kg (6.6-1b) weight loss during the past month. He has smoked 2 packs of cigarettes daily for 32 years. He has hypertension treated with meloprolol. Physical examination shows clubbing of the fingernails. Wheezes are heard over the right upper lobe anteriorly; there are no crackles. Laboratory studies show: Serum: Na+ 125 mEq/L; K+ 3.9 mEq/L; Cl- 85 mEq/L; HCO3- 25 mEq/L; Urea nitrogen 8 mg/dL; Creatinine 0.4 mg/dL; Osmolality 262 mOsmol/kg; Urine osmolality 564 mOsmol/kg A CT scan of the chest shows a 2.4 x 1 2-cm, spiculated noncalcified mass in the right upper lobe eroding into the right bronchus. This patient's laboratory findings are most likely caused by an excess of which of the following hormones? A) ADH (vasopressin) B) Adrenocorticotropic hormone C) Brain natriuretic peptide D) Cortisol E) Thyroxine (T4)

A) ADH (vasopressin)

An 18-year-old man has yellow nodules on the Achilles tendons of his feet and extensor tendons of his hands. Examination of tissue obtained on biopsy of one nodule shows a collection of foamy histiocytes within the dermis. His serum cholesterol concentration is 980 mg/dL, and serum lipoprotein electrophoresis shows a selective increase in LDL. Which of the following is the most likely underlying disorder? A) Absence of functional LDL receptors in hepatocytes B) Defective receptor-binding ß-100 protein in LDL particles C) Diminution of receptor-independent LDL uptake by mononuclear phagocytes D) Increased gastrointestinal absorption of dietary fat E) Partial reduction in functional LDL receptors of hepatocytes

A) Absence of functional LDL receptors in hepatocytes

A 63-year-old man comes to the emergency department 30 minutes after the sudden onset of right-sided chest pain and shortness of breath that began after a 10-hour drive in his truck. The pain is more severe with inspiration. He does not smoke cigarettes. His pulse is 110/min, and respirations are 32/min. The lungs are clear to auscultation. Which of the following findings is most likely on ventilation-perfusion lung scans? A) Absence of perfusion in some areas that are ventilated B) Absence of ventilation in some areas that are perfused C) Absence of ventilation-perfusion mismatching D) Decreased perfusion gradient from the apices to the bases of the lungs E) Increased ventilation-perfusion ratios at lung bases compared with those at the apices

A) Absence of perfusion in some areas that are ventilated

A 35-year-old man is brought to the emergency department because of a 2-hour history of severe fatigue and dizziness, He has had profuse, watery diarrhea for 8 hours despite a lack of oral intake, He recently returned from a medical relief trip to a remote village in Honduras, His temperature is 36.7°C (98°F), pulse is 122/min, and blood pressure is 90(50 mm Hg. Physical examination shows dry skin and decreased capillary refill. Test of the stool for occult blood is negative; the stool is gray and turbid. A Gram stain of the stool shows predominant gram-negative, comma-shaped bacteria; there are no erythrocytes or leukocytes. Which of the following best describes the mechanism of the toxin that caused these findings? A) Activation of adenylyl cyclase B) Activation of guanylyl cyclase C) Activation of Na+—K+ATPase D) Activation of phosphodiesterase E) Deactivation of ADP-ribosylation factors

A) Activation of adenylyl cyclase

A 10-year-old boy who has had type 1 diabetes mellitus for 1 year is receiving insulin. One hour after his morning dose of insulin, he becomes tremulous and diaphoretic and has tachycardia. Several hours later, his symptoms resolve. His blood glucose concentration is now increased. Which of the following is the most likely cause of this patient's hyperglycemia? A) Activation of hepatic adenylyl cyclase B) Activation of muscle glycogen synthase C) Activation of muscle phosphorylase D) Activation of muscle protein phosphatase E) Inhibition of hepatic protein kinase A

A) Activation of hepatic adenylyl cyclase

1. A 47-year-old woman conies to the physician because she wants to "do something about my wrinkles." Physical examination shows fine wrinkles in photodamaged skin on the face. Treatment with tretinoin is started, Collagen synthesis will most likely be increased in this patient by which of the following mechanisms? A) Activation of nuclear gene transcription B) Decreased cAMP production C) Displaced vitamin A from cellular stores D) Increased sebum production E) Protection of keratinocytes from UVB irradiation

A) Activation of nuclear gene transcription

A 35-year-old man comes to the physician because of a 3-year history of an enlarging nose, coarsening of his facial features, muscle weakness, and increased hand and foot size. Physical examination shows a large fleshy nose and prognathism. Serum studies show an increased insulin-like growth factor-I concentration, An MRI of the brain shows a pituitary adenoma. Morphologic analysis of a biopsy specimen of the tumor shows a densely granulated somatotroph adenoma, Further studies show that the Gαs subunit of G proteins in the tumor lack GTPase activity. The tumor cells in this patient most likely have an increased activity of which of the following enzymes? A) Adenylyl cyclase B) Guanylyl cyclase C) Janus kinase D) Phospholipase C E) Tyrosine kinase

A) Adenylyl cyclase

A 19-year-old woman is admitted to the hospital for antibiotic treatment of meningococcal meningitis. She is stabilized, Three days later, her pulse is 120/min, and blood pressure is 60/30 mm Hg. Physical examination shows bilateral flank tenderness. Serum studies show a sodium concentration of 128 mEq/L, potassium of 5.4 mEq/L, and bicarbonate of 20 mEq/L. Which of the following is the most appropriate next step to determine the cause of this patient's hypotension? A) Adrenocorticotropic hormone stimulation test B) Blood culture and antibiotic sensitivity test C) Dexamethasone suppression test D) Thyrotropin-releasing hormone stimulation test E) Urine culture and antibiotic sensitivity test

A) Adrenocorticotropic hormone stimulation test

A previously healthy 32-year-old man is brought to the emergency department because of a 1-day history of intermittent flashing spots and blurred vision, vomiting, confusion, and difficulty walking. He is a painter and says that yesterday he cleaned up a spilled bottle of paint thinner. Ophthalmologic examination shows dilated pupils with hyperemia of the optic disc and retinal edema. Physical examination shows tachypnea. Serum studies show an anion gap metabolic acidosis. The most appropriate pharmacotherapy for this patient is most likely to inhibit the activity of which of the following? A) Alcohol dehydrogenase B) Aldehyde dehydrogenase C) Formaldehyde dehydrogenase D) Lactate dehydrogenase E) Pyruvate dehydrogenase

A) Alcohol dehydrogenase

A previously healthy 25 yer old man is admitted to the hospital for treatment of viral pneumonia. He is mechanically ventilated because of respiratory failure due to hypoxemia. An open-lung biopsy specimen is obtained and shows a pattern of acute lung injury. The presence of which of the following in the biopsy specimen most clearly suggests diffuse alveolar damage rather than bronchiolitis obliterans-organizing pneumonia in this patient? A) Alveolar hyaline membrane B) Foamy macrophages C) Interstitial collagen deposition D) Masson bodies E) Uniform fibroblastic proliferation

A) Alveolar hyaline membrane

A 50-year-old man comes to the emergency department because of a 2-week history of progressive shortness of breath. His pulse is 90/min, respirations are 26/min, and blood pressure is 120/80 mm Hg. Physical examination shows no other abnormalities. Laboratory studies show: Arterial Pco2 30 mm Hg, Arterial Po2 96 mm Hg, Arterial O2 content 12 vol% (N=17%-21%), Mixed venous Po2 36 mm Hg, Mixed venous O2 content 8 vol% (N=10%-16%) Which of the following is the most likely explanation for these findings? A) Anemia B) Drug-induced alveolar hypoventilation C) Residence at a high altitude D) Severe regional mismatching of alveolar ventilation and pulmonary capillary perfusion E) Voluntary hyperventilation

A) Anemia

The following blood gas values are measured in a resting patient: Arterial Po2 96 mm Hg Arterial Pco2 40 mm Hg Arterial O2 content 12.0 vol% (N=20) Mixed venous Po2 36 mm Hg (N=40) Mixed venous O2 content 8.0 vol% (N=15). These values are most consistent with which of the following? A) Anemia B) Drug-induced alveolar hypoventilation C) Residence at high altitude D) Severe regional mismatching of alveolar ventilation and pulmonary capillary perfusion E) Voluntary hyperventilation

A) Anemia

A 40-year-old woman comes to the physician for an initial examination. She says that she is health conscious and eats only natural organic foods. Fasting serum laboratory studies show a triglyceride concentration of 380 mg/dL; a complete blood count and serum electrolyte concentrations are within the reference ranges. The physician suggests that the patient try taking fish oil and a supplement to treat her dyslipidemia. Which of the following best describes the mechanism of action of the most appropriate vitamin for this patient? A) Antagonizes VLDL-cholesterol secretion B) Increases catabolism of LDL-cholesterol C) Inhibits cholesterol uptake D) Inhibits HMG CoA E) Stimulates PPAR-α receptors

A) Antagonizes VLDL-cholesterol secretion

A 22-year-old woman comes to the physician for a follow-up examination_ One year ago, she was diagnosed with a pulmonary embolism. Two years ago, she delivered a female stillborn at 23 weeks' gestation. Physical examination today shows no abnormalities. Laboratory studies show a platelet count of 250,000/mm3, a prothrombin time within the reference range, and an increased partial thromboplastin time. The findings in this patient are most consistent with which of the following conditions? A) Antiphospholipid antibody syndrome B) Factor V Leiden mutation C) Increased factor VIII (antihemophilic factor) concentration D) Protein C deficiency E) Prothrombin G2O210A mutation

A) Antiphospholipid antibody syndrome

A healthy 56-year-old man comes to the physician for a pre-employment physical. Cardiac examination shows a systolic heart murmur that is best heard in the second interspace at the right sternal border and is transmitted to the carotid arteries. The murmur begins immediately after S1, rises in crescendo before falling in pitch, and ceases before S2. Which of the following is the most likely cause of the murmur? A) Aortic stenosis B) Atrial septal defect C) Mitral regurgitation D) Pulmonary regurgitation E) Tricuspid stenosis

A) Aortic stenosis

A 25 year old woman undergoes a radical mastectomy with removal of the axillary lymph nodes on the left side. After the operation, she develops painful edema of the left upper extremity. Which of the following therapies is most likely to decrease the swelling in this patent?

A) Application of heat B) Compression sleeve C) Diuretic therapy 0) Exercise therapy E) Passive movement B) Compression sleeve

A 2-year-old girl is brought to the emergency department 20 minutes after she accidentally ingested insecticide. Her father reports that he was spraying his lawn and accidentally left the open bottle on the ground. His daughter walked over, picked up the bottle, and started to drink, but then she immediately threw down the bottle. She is in moderate respiratory distress. Her respirations are 18/min and shallow. Pulse oximetry on room air shows an oxygen saturation of 82%. Physical examination shows copious oral secretions, In addition to securing an airway, the most appropriate immediate step is administration of which of the following medications? A) Atropine B) Diphenhydramine C) Physostigmine D) Pralidoxime E) Succinylcholine

A) Atropine

A 35-year-old man who is a farmer is brought to the hospital by his wife because of difficulty breathing, sweating, excess salivation, and diarrhea for the past 2 hours. His wife says that the symptoms started when he was applying a new insecticide to his crops. Temperature is 36°C (96.8°F), pulse is 50/min, respirations are 22/min, and blood pressure is 90/60 mm Hg. Which of the following is the most appropriate treatment? A) Atropine B) Ipratropium C) Mecamylamine D) Neostigmine E) Propantheline

A) Atropine

A 67-year-old man comes to the physician for a follow-up examination, Eight months ago, he received a diagnosis of primary lung carcinoma involving the adrenal glands, liver, and bone. He has had an 8-kg (17.6-lb) weight loss during the past 3 months, Physical examination shows cachexia and significant muscle wasting. Which of the following intracellular components is most likely increased in this patient's muscle cells? A) Autophagic vacuoles B) Endoplasmic reticulum C) Golgi complex D) Mitochondria E) Mitotic spindles

A) Autophagic vacuoles

40. A 64-year-old man comes to the physician because of a 3-day history of fever and cough productive of purulent sputum. He has smoked 2 packs of cigarettes daily for 30 years. His temperature is 39.4°C (103°F). Physical examination shows erythematous tympanic membranes. Rhonchi are heard on auscultation of the chest. A chest x-ray shows consolidation in the right middle lobe. The physician wants to prescribe a drug that inhibits protein synthesis by binding to the 50S subunit of bacterial ribosomes. Which of the following drugs should be prescribed for this purpose? A) Azithromycin B) Cefdinir C) Cefepime 0) Levofloxacin E) Trimethoprim-sulfamethoxazole

A) Azithromycin

A 78-year-old man comes to the physician because of fever, chills, fatigue, shortness of breath, and a 5.4-kg (12-lb) weight loss over the past month. He underwent urinary catheterization 6 weeks ago for prostatic obstruction, His temperature is 38.5°C (101.3°F), pulse is 100/min, respirations are 14/min, and blood pressure is 160/80 mm Hg, Examination of the chest shows a soft S1 and a normal S2. A grade 2/6 early diastolic murmur is maximal in the second left intercostal space and is accentuated when the patient leans forward with held expiration. The lungs are clear, Which of the following is the most likely diagnosis? A) Bacterial endocarditis B) Peritonitis C) Prostatitis D) Pulmonary embolus E) Viral pneumonia

A) Bacterial endocarditis

An investigator is studying epithelial repair in the small intestine of an experimental animal. The plan is to identify the location of the most active cell division. This cell activity is most likely to be found in which of the following regions? A) Base of the crypt B) Brunner glands C) Peyer patches D) Top of the villi

A) Base of the crypt

21. A 68-year-old man has a serum creatinine concentration of 2.3 mg/dL due to chronically increased hydrostatic pressure in Bowman space. Which of the following disorders is the most likely cause of these findings? A) Benign prostatic hyperplasia B) Congestive heart failure C) Hypertension D) Interstitial nephritis E) Nephrotic syndrome F) Type 2 diabetes mellitus

A) Benign prostatic hyperplasia

An 18-year-old woman is brought to the emergency department 2 hours after she ingested approximately 100 aspirin tablets in a suicide attempt. Physical examination shows tachypnea. Two days after admission to the hospital, her hemoglobin concentration is 12 g/dL. Test of the stool for occult blood is positive. Which of the following additional hematologic findings is most likely abnormal in this patient? A) Bleeding time B) Fibrin degradation products C) Partial thromboplastin time D) Platelet count E) Prothrombin time

A) Bleeding time

8. A 48-year-old woman comes to the physician because of a 2-month history of urinary frequency and urgency. Physical examination shows no abnormalities. The physician prescribes a medication that will affect the normal stimulation of bladder contraction. Which of the following is the most likely effect of this drug? A) Blockade of parasympathetic innervation of the detrusor muscle B) Blockade of postganglionic sympathetic innervation of the bladder trigone C) Blockade of preganglionic parasympathetic transmission D) Enhancement of afferent feedback from the bladder stretch receptors E) Enhancement of peristalsis of the ureters F) Enhancement of sympathetic preganglionic transmission

A) Blockade of parasympathetic innervation of the detrusor muscle

Two days after undergoing a right hip replacement operation, a 67-year-old woman has shortness of breath and pain on the right side of her back with deep breathing. She appears anxious. Her pulse is 100/min. and respirations are 24/min. A pleural friction rub is heard in the right lung base. Examination of the right calf shows erythema and induration. Immediate treatment with oxygen, 4L/min via nasal cannula, is initiated. Arterial blood gas analysis shows: pH 7.54, Pco2 29 mm Hg, Po2 61 mm Hg. CT angiography of the chest shows a large embolism occluding blood flow to the right lower lobe. Which of the following terms best describes the current ventilation-perfusion relationship in the right lower lobe of this patient? A) Dead space B) Diffusion abnormality C) Hypoventilation D) Low Fio2 E) Shunt

A) Dead space

A previously healthy 35-year-old man comes to the physician because of a 2-year history of gradually decreasing libido and intermittent inability to achieve erection or orgasm; he also has had decreased growth of facial hair during this period, Puberty began at the age of 12 years, He takes no medications. He does not smoke or drink alcohol. He is 180 cm (5 ft 11 in) tall and weighs 75 kg (165 lb); BMI is 23 kg/m2. His vital signs are within normal limits. Physical examination shows a typical male-pattern distribution of hair. Genital examination shows no abnormalities. Serum studies show a thyroid-stimulating hormone concentration of 2 µU/mL, prolactin concentration of 120 ng/mL, and total testosterone concentration of 25 nmol/L (N=10-35). Semen analysis shows a markedly decreased sperm count with impaired motility. An MRI of the brain shows a 1-cm lesion in the pituitary gland. Administration of which of the following is the most appropriate treatment for this patient? A) Bromocriptine B) Gonadotropin-releasing hormone C) Human chorionic gonadotropin D) Sildenafil E) Testosterone F) Thyroxine

A) Bromocriptine

A 45-year-old woman has a thyroidectomy because of asymmetric enlargement of the thyroid gland, first noticed during a physical examination 6 weeks ago, She underwent an adrenalectomy for pheochromocytoma 3 years ago, Microscopically, the bilateral thyroid lesions are composed of spindle cells arranged in small clusters. Deposits of amyloid are present between neoplastic cells. Foci of C-cell hyperplasia are also present. The lesions are confirmed as malignant. Which of the following markers is most appropriate to monitor for the development of recurrence of the thyroid neoplasm? A) Calcitonin B) α-Fetoprotein C) Human chorionic gonadotropin D) Norepinephrine E) Thyroid-stimulating hormone

A) Calcitonin

A 42-year-old woman conies to the physician because of a 1-month history of abdominal pain, especially after eating fatty meals, She is 180 cm (5 ft 11 in) tall and weighs 100 kg (220 lb): BMI is 31 kg/m2. Physical examination shows jaundice and tenderness of the right upper quadrant of the abdomen. An increase in which of the following liver functions is most likely in this patient? A) Cholesterol synthesis B) Conjugation of bilirubin C) Deamination of amino acids D) Glycogen synthesis E) Secretion of angiotensinogen

A) Cholesterol synthesis

12. A 32-year-old man begins to laugh while eating dinner with his friends. A small particle of food irritates his larynx and provokes him to cough. Which of the following best describes the position of his vocal cords throughout this sequence? While Swallowing; Immediately After Laryngeal Irritation; While Coughing A) Closed; closed; open B) Closed; open; closed C) Closed; open; open D) Open; closed; closed E) Open; closed; open F) Open; open; closed

A) Closed; closed; open

11. A 22-year-old woman is admitted to the hospital because of a 10-day history of polydipsia and polyuria. She says that the urge to urinate often awakens her at night. She has been taking lithium carbonate for 2 years for bipolar disorder; her dosage was increased 6 months ago because of recurrent severe manic episodes. Her vital signs are within normal limits. Physical examination shows no abnormalities. Over the next 24 hours, urine excretion totals 6.5 L. Laboratory studies at this time show a serum sodium concentration of 148 mEq/L, serum osmolality of 315 mOsmol/kg, and urine osmolality of 75 mOsmol/kg. After administration of desmopressin. urine output and osmolality do not change. The most likely cause of the polyuria in this patient is a defect in which of the following sites in the nephron? A) Collecting duct B) Glomerular capillary C) Juxtaglomerular apparatus D) Loop of Henle E) Proximal tubule

A) Collecting duct

49. A 76-year-old man is brought to the emergency department by his brother 30 minutes after the onset of severe substernal chest pain that began while they were having an argument. Administration of three sublingual nitroglycerin tablets has not relieved the pain. He has a 5-year history of ischernic heart disease His pulse is 110/min. respirations are 20/min. and blood pressure is 120/80 mm Hg. Cardiac examination shows an S1 and S2 without murmurs, gallops, or rubs. An ECG shows ST-segment elevations in leads I, aVL, and V4 through V6. Which of the following best describes the role of the argument in the development of this patient's chest pain? A) Coronary vasospasm due to α1-adrenergic stimulation B) Coronary vasospasm due to ß1-adrenergic stimulation C) Decreased preload for the left ventricle due to a α1-adrenergic stimulation D) Increased afterload for the left ventricle due to ß1-adrenergic stimulation E) Negative chronotropic effect of a α1-adrenergic stimulation F) Positive inotropic effect of ß2-adrenergic stimulation

A) Coronary vasospasm due to α1-adrenergic stimulation

A previously healthy 27 year old man comes to the physician because he has had a cough productive of blood-tinged sputum for 4 days. His blood pressure is 160/100 mm Hg. His serum creatinine concentration is 4.8 mg/dL. Urinalysis shows erythrocyte casts. Immunofluorescence microscopy of a renal biopsy specimen shows linear deposits of IgG in the glomeruli. Which of the following histologic abnormalities in the glomerulus is most likely? A) Crescent formation B) Focal segmental change C) Global sclerosis D) Membranoproliferative changes E) Minimal change F) Nodular glomerulosclerosis

A) Crescent formation

24. An investigator is developing a new drug that is designed to stimulate epithelial cell turnover in the small intestine, To be successful. this drug must target which of the following structures? A) Crypt cells B) Folds of Kerckring C) Goblet cells D) Mature enterocytes E) Microvilli

A) Crypt cells

A 73 year old man has had a poor appetite and has lost 11.5 kg (25 lb) over the past 4 months. He has been otherwise asymptomatic. Laboratory evaluation shows normochromic nomocytic anemia. An x-ray of the chest shows a 2-cm perihilar mass. Biopsy of the mass shows small cell carcinoma of the lung. Physical examination and imaging studies show no clinically detectable metastases. Which of the following is the most likely explanation for the weight loss? A) Cytokine effect B) Infection C) Renal failure D) Superior vena cava syndrome E) Tumor hormone production

A) Cytokine effect

A 35-year-old woman is brought to the emergency department by her husband because she passed out while washing dishes. The patient says she began to feel weak and dizzy but does not remember anything else_ While the nursing staff is checking her vital signs and drawing blood, the husband tells the physician that the patient has bulimia nervosa and that she has been binging more frequently over the past 3 weeks. He suspects that she has been vomiting. Which of the following patterns of abnormal laboratory test results most likely indicates recurrent vomiting in this patient? K+ HCO3- Anion Gap pH A) DOWN UP normal UP A) DOWN DOWN normal DOWN A) DOWN DOWN UP DOWN A) UP DOWN UP DOWN A) UP DOWN normal UP

A) DOWN UP normal UP

A 25-year-old man who has a history of facial flushing and rapid heal rate each time he consumes a small amount of alcohol participates in a study of alcohol intolerance. Molecular analysis shows the presence of a lysine (K487, oriental variant) for glutamate (E487, native variant) substitution in aldehyde dehydrogenase. The kinetic characteristics of the enzyme variants are shown in the table. Enzyme KM (NAD+) (µM) Kcat (min^-1) E487 37 180 K487 5600 9.5 Based on these findings, which of the following is the most likely cause of this man's condition? A) Decreased catalytic efficiency of K487 B) Increased catalytic efficiency of E487 C) Less rapid turnover by E487 D) More rapid turnover by K487 E) Tighter binding of NAD+ to K487 F) Weaker binding of NAD+ to E487

A) Decreased catalytic efficiency of K487

A 22-year-old man comes to the physician because of an 8-year history of episodes of yellow-tinged eyes. He is a medical student, and he says that he has noticed that the symptom occurs during studying for final exams and other periods of intense stress. He has had no pain. Physical examination shows mild scleral icterus. Serum studies show: Bilirubin, total 3.2 mg/dL, Direct 0.4 mg/dL, Alkaline phosphatase 35 U/L, AST 16 U/L, ALT 15 U/L Which of the following best explains this patient's signs? A) Decreased conjugation of bilirubin B) Hepatitis C virus C) Increased hepatic copper concentration D) Intermittent obstruction of the common bile duct E) Surreptitious acetaminophen abuse

A) Decreased conjugation of bilirubin

A 48-year-old woman comes to the physician because of a 2-month history of fatigue and intermittent headaches, Her blood pressure is 180/110 mm Hg. Physical examination shows no other abnormalities. Serum studies show a decreased potassium concentration and increased aldosterone concentration, A CT scan of the abdomen shows a tumor on the adrenal gland. Which of the following additional findings in this patient most likely supports the diagnosis of an aldosterone-secreting adrenal adenoma? A) Decreased plasma renin activity B) Decreased serum sodium concentration C) Decreased urine potassium concentration D) Increased plasma renin activity E) Increased serum sodium concentration F) Increased urine metanephrine concentration

A) Decreased plasma renin activity

A 36-year-old man undergoes elective liposuction under general anesthesia, The operation is terminated prematurely when the anesthesiologist notices that the patient has developed hyperthermia, tachycardia, and marked muscle rigidity. The drug that should be administered has which of the following actions? A) Decreases release of Ca2+ from the sarcoplasmic reticulum B) Enhances renal tubular excretion of Ca 2+ C) Increases plasma Ca2+ concentration D) Opens cellular membrane Ca2+ and Na+ channels E) Shifts Ca2+ from the extracellular to the intracellular space

A) Decreases release of Ca2+ from the sarcoplasmic reticulum

20. An investigator studying the function of a voltage-gated sodium channel develops a mutant form of the channel that inactivates more rapidly than normal. Which of the following is the most likely effect of this mutation on the electrical properties of the neuron? A) Decreases the amplitude of the action potential B) Decreases the maximum frequency of action potential production C) Increases the conduction velocity of the axon D) Increases the input resistance of the axon E) Increases the rate of depolarization of the action potential

A) Decreases the amplitude of the action potential

47. While scuba diving, a 46-year-old man ascends without appropriate decompression. As part of the developing decompression sickness, small gas bubbles form in the spinal vessels supplying the dorsal white matter of the upper thoracic spinal cord. The most likely neurologic outcome is loss of which of the following? A) Discriminative sensation in the lower extremities B) Motor and reflex activity and all sensation in the lower extremities C) Motor and reflex activity in the lower extremities D) Motor and reflex activity in the upper extremities E) Pain and temperature sensation in the upper extremities

A) Discriminative sensation in the lower extremities

17. An 83-year-old man is brought to the emergency department after being found at home bedridden and confused. He takes no medications. Temperature is 35.66°C (96°F), pulse is 100/min, and blood pressure is 85/50 mm Hg. Blood pressure is unchanged after intravenous infusion of 1 liter of isotonic saline. A pulmonary artery catheter is inserted and the following findings are obtained: Cardiac output high, Pulmonary capillary wedge pressure low, Systemic vascular resistance low Which of the following is the most likely cause of the hypotension? A) Early septic shock B) Gastrointestinal bleeding C) Hypothyroidism D) Massive pulmonary embolism E) Silent myocardial infarction

A) Early septic shock

32. A previously healthy 35-year-old woman comes to the physician because of a 3-month history of progressive shortness of breath with exertion. Her respirations are 26/min. Physical examination shows jugular venous distention with a prominent a wave, The lungs are clear to auscultation. Cardiac examination shows a loud pulmonic component of S2 and a right-sided S4 gallop. This patient's condition most likely involves an increased pulmonary expression of which of the following? A) Endothelin-1 B) Nitric oxide C) ProstacycEin (PGI2) synthase D) Thrombomodulin E) Voltage-gated potassium channels

A) Endothelin-1

A 67-year-old man comes to the physician because of a 3-month history of fatigue and shortness of breath. He has not been examined by a physician in several years_ His pulse is 90/min, respirations are 15/min, and blood pressure is 150/98 mm Hg. Physical examination shows conjunctival pallor. Laboratory studies show: Hemoglobin 8.5 g/dL, Hematocrit 26%, Mean corpuscular volume 90 µm^3, Red cell distribution width 14.4% (N=13% - 15%). Serum: Creatinine 2.9 mg/dL, Ferritin 144 ng/mL, Iron 24 µg/dL, Transferrin saturation 23% (N=20%-50%). In addition to iron supplementation. which of the following is the most appropriate therapy for this patient? A) Erythropoietin B) Folic acid C) Granulocyte colony-stimulating factor D) Transfusion of red blood cells E) Vitamin B12 (cyanocobalamin)

A) Erythropoietin

41. A 52-year-old man comes to the physician because of a 3-week history of increased thirst and urinary frequency; he has had a 4.5-kg (10-1b) weight loss during this period. He has hypertension and hyperlipidemia treated with pravastatin and metoprolol, He appears mildly ill, He is 180 on (5 ft 11 in) tall and weighs 109 kg (240 lb); BMI is 34 kg/m2. His pulse is 94/min, respirations are 16/min, and blood pressure is 130/80 mm Hg. Examination shows no other abnormalities. Which of the following is most likely to be increased in this patient? A) Fasting serum glucose concentration B) Glycogen synthesis in muscle C) Insulin-mediated glucose disposal rate D) Serum calcium concentration E) Serum HDL-cholesterol concentration F) Urine sodium concentration

A) Fasting serum glucose concentration

A 41-year-old woman is evaluated because of increasingly severe headaches for 6 weeks. Her blood pressure is 163/100 mm Hg while standing and supine A bruit is heard over the left costovertebral angle. Urinalysis shows no abnormalities. An angiogram of the left renal artery shows alternating areas of stenosis and aneurysmal dilatation ('string of beads' sign). Which of the following conditions of the renal artery is the most likely diagnosis? A) Fibromuscular dysplasia B) Hyaline arteriolosclerosis C) Intimal fibroplasia D) Periarterial fibroplasia E) Perimedial hyperplasia

A) Fibromuscular dysplasia

2. During an experiment, it is found that instillation of lipid into the duodenum leads to the release of a hormonal substance from endocrine cells in the duodena mucosa. Which of the following best describes the major physiologic action of this substance? A) Gallbladder contraction B) Increased gastric contractility C) Pancreatic endocrine stimulation D) Salivary gland stimulation E) Splanchnic vasoconstriction

A) Gallbladder contraction

A 23-year-old man comes to the physician because of a low-grade fever, nasal congestion, and muscle aches for 1 week. He states that his girlfriend recently told him that his eyes appeared yellow. He has not had clay-colored stools or dark urine, His only medication is ibuprofen, He has no recent history of foreign travel, tattoos. use of intravenous drugs, or exposure to blood products. He is 183 cm (6 ft) tall and weighs 77 kg (170 lb); BMI is 23 kg/m2, Physical examination shows mild scleral icterus but no subungual icterus. The remainder of the examination shows no abnormalities. Serum studies show: Bilirubin, total 2.6 mg/dL Direct 0.3 mg/dL Alkaline phosphatase 30 U/L AST 12 U/L ALT 10 U/L Which of the following is the most likely diagnosis? A) Gilbert syndrome B) Hepatitis D C) Hereditary spherocytosis D) Idiopathic cirrhosis E) Steatohepatitis

A) Gilbert syndrome

A 23-year-old woman comes to the physician because of a 3-week history of frequent thirst and urination; she also has had a 3-kg (6.6-lb) weight loss during this period. Physical examination shows dehydration and tachypnea, Serum studies show a glucose concentration of 330 mg/dL, 2+ ketones, and ph of 7.2. Following the administration of intravenous fluids and insulin, there is marked improvement, The activity of which of the following enzymes has most likely increased in this patient's hepatocytes because of this treatment? A) Glucokinase B) Glucose 6-phosphatase C) Glycogen phosphorylase D) Phosphoenolpyruvate carboxykinase E) Phosphorylase kinase

A) Glucokinase

A 22-year-old nulligravid woman comes to the physician for counseling prior to conception. Menses have occurred at irregular 26- to 32-day intervals since menarche at the age of 14 years. She is 163 cm (5 ft 4 in) tall and weighs 85 kg (187 lb): BMI is 32 kg/m2. Her blood pressure is 130/80 mm Hg. Physical examination shows mild hirsutism and velvety brown, thickened skin at the base of the neck and around the axillae. This patient most likely also has which of the following conditions? A) Hyperinsulinemia B) Hyperthyroidism C) Hypoandrogenism 0) Hypoglycemia E) Hypoprolactinemia

A) Hyperinsulinemia

A 25-year-old woman comes to the physician because of a 6-month history of irregular menstrual periods and progressive acne, Menses have occurred at irregular 25- to 42-day intervals. She is 1 I70 cm (5 ft 7 in) tall and weighs 100 kg (220 lb) BMI is 35 kg/m2. Her voice is deep. Physical examination shows dark hair over the upper lip and chin, and open and closed comedones over the face, chest, and back. Serum studies are most likely to show which of the following in this patient? Fasting Insulin; Testosterone; Luteinizing Hormone A) Increased; increased; increased B) Increased; increased; decreased C) Increased; decreased; increased D) Decreased; increased; increased E) Decreased; increased; decreased F) Decreased; decreased; decreased

A) Increased; increased; increased

A 19-year-old man is admitted to the hospital following a motor vehicle collision. Physical examination shows a penetrating wound to the right cerebral cortex with complete paralysis of the left lower extremity, fracture of the right mid humerus with severing of the radial nerve, and a fracture of the right tibia. Treatment includes cast immobilization of the right upper extremity and right knee and ankle. After 10 weeks, the casts are scheduled to be removed from the right upper and lower extremities. At this point, the deep tendon reflex is most likely to be strongest in which of the following locations in this patient? A) Left Achilles tendon B) Left biceps tendon C) Right brachioradialis tendon D) Right patellar tendon

A) Left Achilles tendon

A 38-year-old man is brought to the emergency department 30 minutes after being found near his home unable to stand upright. He appears lethargic. His pulse is 110/min, and blood pressure is 90/62 mm Hg while supine. Physical examination shows dry mucous membranes and poor skin turgor. Abdominal examination shows midepigastric tenderness. Laboratory studies show: Serum: Na+ 143 mEq/L, K÷ 3.2 mEq/L, Cl- 101 mEq/L, HCO3- 11 mEq/L. Arterial blood gas analysis on room air: pH 7.28, Pco2 23 mm Hg, Po2 98 mm Hg. Which of the following is the most likely current acid-base status in this patient? A) Metabolic acidosis B) Metabolic alkalosis C) Mixed metabolic acidosis and metabolic alkalosis D) Mixed respiratory acidosis and metabolic alkalosis E) Mixed respiratory acidosis and respiratory alkalosis F) Mixed respiratory alkalosis and metabolic alkalosis G) Respiratory acidosis H) Respiratory alkalosis

A) Metabolic acidosis

A 49-year-old woman with breast cancer develops increased numbness of both hands and feet during the 3-week interval between her third and fourth rounds of chemotherapy. Sensation to pinprick and fine touch is decreased over the hands, wrists, ankles, and feet. The physician suspects that this distribution of sensory loss is most likely an adverse effect of chemotherapeutic agents. Which of the following is most likely disrupted in this patient as a result of the chemotherapy? A) Microtubules for axonal transport B) Myelin sheaths for saltatory conduction of action potentials C) Neurofilaments for structural support of axons D) Sodium channels for membrane depolarization E) Synaptic vesicles for synaptic transmission

A) Microtubules for axonal transport

A 38-year-old woman comes to the physician because of a 10-week history of epigastric pain and frequent stools, She has had two episodes of renal calculi during the past 2 years. Physical examination shows multiple superficial lipomata and mild epigastric tenderness, Upper endoscopy shows two large, nonbleeding duodenal ulcers. Her gastric pH is 2.3, and serum gastrin concentration obtained immediately after the procedure is 2000 pg/mL (N<100). Proton pump inhibitor therapy is begun. Measurement of which of the following is the most appropriate next step in management? A) Serum calcium concentration B) Serum cortisol concentration C) Serum tissue transglutaminase activity D) Stool α1-antitrypsin concentration E) Urine 5-hydroxyindoleacetic acid concentration

A) Serum calcium concentration

A 72-year-old man with chronic obstructive pulmonary disease comes to the physician because of progressive shortness of breath during the past 3 months. He had smoked 1 1/2 packs of cigarettes daily for 52 years, but he recently decreased the amount to 1 pack daily. He is 178 cm (5 ft 10 in) tall and weighs 109 kg (240 lb); BMI is 34 kg/m2. His pulse is 80/min. respirations are 22/min. and blood pressure is 130/80 mm Hg. Pulse oximetry on room air shows an oxygen saturation of 94%. Coarse expiratory rhonchi are heard on auscultation of the chest. Breath sounds are decreased bilaterally. Which of the following interventions is most likely to slow the rate of decline in this patient's pulmonary function? A) Smoking cessation B) Tiotropium therapy C) Pulmonary rehabilitation D) ß2-Adrenergic agonist therapy E) Corticosteroid therapy

A) Smoking cessation

31. A 43-year-old woman with metastatic colon cancer has maintained remission by taking bevacizumab as a single agent. The ability to administer this agent despite it being a foreign protein is most likely a result of which of the following? A) The agent being a humanized antibody B) The agent containing the variable region of a marine antibody molecule directed against vascular endothelial growth factor C) B-cell dysfunction D) The irrimunosuppressive effect of previously administered chemotherapy E) T-cell dysfunction

A) The agent being a humanized antibody

49. A 17-year-old boy is brought to the emergency department on a hot summer day after he collapses while working as a traffic controller for a road construction crew. When he was 4 years old_ he was treated with numerous skin grafts after sustaining full-thickness burns involving 30% of his body surface area. Examination shows hyperthermia. Which of the following is the most likely explanation for this occurrence? A) The burn scars and skin grafts lack eccrine glands, limiting his ability to sweat B) Central thermoregulation is inadequate C) Injured skin is more likely to release endogenous pyrogens increasing core temperature D) Skin grafts and scars are more sensitive to heat and sunlight

A) The burn scars and skin grafts lack eccrine glands, limiting his ability to sweat

41. A healthy 25-year-old man undergoes an exercise stress test. Which of the following sets of physiologic changes is most likely to occur in this man's exercising skeletal muscles? Arteriolar Diameter Tissue Adenosine Concentration Vascular Conductance A) UP UP UP B) UP UP DOWN C) UP DOWN UP D) UP DOWN DOWN E) DOWN UP UP F) DOWN UP DOWN G) DOWN DOWN UP H) DOWN DOWN DOWN

A) UP UP UP

A 45-year-old man comes to the physician for a preemployment examination. He has a 10-year history of hypertension and does not adhere to his medication regimen. His temperature is 37C (98.6F), pulse is 80/min, respirations are 14/min, and blood pressure is 160/100 mm Hg. Cardiac examination shows that the apical impulse is displaced laterally. A loud S2 and and S4 gallop are heard on auscultation, Echocardiography shows thickening of the left ventricular wall. The most likely mechanism causing the changes in this patient's left ventricle is best represented by which of the following sets of findings in cardiac muscle? Transcription Factor c-Jun p-Myosin Heavy Chain Endothelin A) UP UP UP A) UP DOWN UP A) UP DOWN DOWN A) DOWN UP UP A) DOWN UP DOWN A) DOWN DOWN UP

A) UP UP UP

A 63-year-old man comes to the physician because of an abnormal blood pressure reading taken during a health fair 1 week ago. He has no history of major medical illness. At a previous examination 6 months ago, his blood pressure was 135/85 mm Hg. His blood pressure today is 170/98 mm Hg. A bruit is heard over the left renal artery. Spiral CT angiography shows left arterial stenosis, Which of the following sets of changes is most likely in this patient? Total Peripheral Resistance Plasma Renin Activity Serum Aldosterone Concentration A) UP UP UP B) UP UP DOWN C) UP DOWN UP D) UP DOWN DOWN E) DOWN UP UP F) DOWN UP DOWN G) DOWN DOWN UP H) DOWN DOWN DOWN

A) UP UP UP

A 24-year-old man comes to the physician because of nasal congestion, a watery nasal discharge, and sneezing. He says that these symptoms happen each spring, when grass and tree pollens are abundant. Which of the following types of drugs is most likely to be effective in relieving these symptoms in the short term? A) α-Adrenergic agonist B) α-Adrenergic antagonist C) ß-Adrenergic agonist D) ß-Adrenergic antagonist E) Nicotinic cholinergic agonist F) Nicotinic cholinergic antagonist

A) α-Adrenergic agonist

27. A 17-year-old boy has septic shock that is unresponsive to intravenous ADI-1(vasopressin) therapy. This treatment is discontinued, and high-dose dopamine therapy is initiated. Stimulation of which of the following receptors is most likely to be of benefit to this patient? A) α1-Adrenoreceptors B) α2-Adrenoreceptors C) ß1-Adrenoreceptors D) ß2-Adrenoreceptors E) D2-receptors

A) α1-Adrenoreceptors

A healthy 70-year-old woman has participated in a longitudinal study of the effects of aging on performance during pulmonary function tests for the past 50 years. She has undergone a complete set of tests, including arterial blood gas analyses, every 5 years. Which of the following sets of changes best represents this woman now compared with her results at the age of 20 years? Residual Volume; Arterial Po2; Alveolar-arterial Po2 Difference A) UP; UP; DOWN B) UP; DOWN; UP C) NO CHANGE; UP; UP D) NO CHANGE; UP; DOWN E) DOWN; UP; DOWN F) DOWN; DOWN; DOWN

B) UP; DOWN; UP

A male newborn delivered at 28 weeks' gestation is tachypneic and hypoxemic. Which of the following altered structure changes in the type II pneumocytes is the mast likely cause of this patient's condition? A) Abundant rough endoplasmic reticulum B) Decreased numbers of lamellar bodies C) Distinct junctional complexes with adjacent type I pneumocytes D) Numerous microvilli E) Reticular fibers

B) Decreased numbers of lamellar bodies

An 18-year-old woman develops sepsis after an abortion, Within 24 hours she becomes dyspneic, oliguric, and develops petechiae, ecchymoses, and bleeding from venipuncture sites. Which of the following is the laboratory finding most consistent with this disease process? A) Decreased bleeding time B) Decreased plasma fibrinogen concentration C) Increased antineutrophil antibody titers D) Increased plasma factor VIII (antihemophilic factor) concentration E) Increased plasma hemolytic complement activity

B) Decreased plasma fibrinogen concentration

A 6-year-old boy with severe mental retardation and choreoathetosis is being evaluated for Lesch-Nyhan syndrome. Leukocytes are obtained for an assay of hypoxanthine-guanine phosphoribosyltransferase at saturating concentrations of phosphoribosylpyrophosphate and varying concentrations of guanine. Results of the data are shown. Guanine Velocity (Patient) (mM) µmoles/min 0.06 0.26 0.2 0.60 0.5 1.1 1.0 1.4 2.0 1.7 4.0 1.8 8.0 1.9 20 2.0 200 2.0 Which of the following is the approximate Michaelis constant [Km] (in mM) for guanine in this patient? A) <0.06 B) 0.2 to 05 C) 0.5 to 1.0 D) 1.0 to 2.0 E) 8.0 to 20

B) 0.2 to 05

A 45-year-old man comes to the physician for a follow-up examination 1 week after he underwent an emergency appendectomy. Physical examination shows mild scleral icterus and a well-healing surgical incision. Laboratory studies show: Hemoglobin 12.6 g/dL, Hematocrit 40%. Serum: Bilirubin, total 3.2 mg/dL, Direct 0.2 mg/dL, Indirect 3 mg/dL, AST 32 U/L, ALT 28 U/L Which of the following best explains these findings? A) Cholelithiasis B) Gilbert syndrome C) Hepatitis D) Liver failure E) Surgical bile duct trauma

B) Gilbert syndrome

A healthy 25-year-old man participates in a study of muscle function. The electrophysiologic observations made on a muscle biopsy specimen are shown. Via iontophoresis, 1 µM acetylcholine (ACh) was applied to the muscle surface. Extracellular Ca2+ concentration was decreased to prevent end-plate potentials from acting as a suprathreshold for muscle action potentials. epp (in mV); Amplitudemepp (in mV); Amplitude Response (in mV) to ACh Normal muscle: 10; 1; 1 epp=end-plate potential; mepp= miniature epp Based on these findings, which of the following electrophysiologic characteristics is expected in a muscle biopsy specimen from a patient with acute botulism? epp Amplitude(in mV); mepp Amplitude(in mV); Response (in mV) to ACh (1µM) A) 1; 0.5; 0.5 B) 1; 1; 1 C) 2; 0.2; 0.2 D) 5; 0.2; 0.1 E) 15; 1; 1

B) 1; 1; 1

A 54-year-old man has an aneurysm in the distal portion of his abdominal aorta, The estimated cross-sectional area of the aneurysm is 2 cm2 and the mean velocity of blood flow through the aneurysm is 20 cm/sec, Which of the following best represents the flow rate (in L/min) through the aneurysm? A) 1.0 B) 2.4 C) 3.2 D) 3.6 E) 4.0

B) 2.4

A 65-year-old man with a history of ischemic heart disease comes to the physician because of fatigue and pain and swelling in his legs. His blood pressure is 135/80 mm Hg and pulse is 80/min. End-systolic volume is 80 mL and end-diastolic volume is 120 mL. Which of the following best represents the ejection fraction in this man? A) 25% B) 33% C) 36% D) 40% E) 50% F) 67%

B) 33%

A 32-year-old man is brought to the emergency department in a coma. Drug overdose is suspected. Pulse is 68/min, respirations are 8/min and shallow, and blood pressure is 130/78 mm Hg. Lung fields are clear on percussion and auscultation. Heart rate is regular with normal heart sounds. Which of the following sets of arterial blood values determined while the patient is breathing room air is most likely? pH; Po2(mm Hg); Pco2 (mm Hg); HCO3-(mm Hg) A) 7.15; 30; 30; 10 B) 7.28; 50; 50; 27 C) 7.40; 30; 40; 24 D) 7.40; 80; 70; 42 E) 7.50; 80; 30; 24

B) 7.28; 50; 50; 27

10. During a series of normal skeletal muscle twitches, the ATP concentration does not fall appreciably because of which of the following? A) ATP is hydrolyzed only during relaxation B) ATP is quickly regenerated from creatine phosphate C) Creatine phosphate is hydrolyzed by myosin D) The initial ATP concentration is high

B) ATP is quickly regenerated from creatine phosphate

48. A 35-year-old man who uses crack cocaine daily comes to the emergency department because of a 2-hour history of substernal chest pain, He last used cocaine 6 hours ago_ His temperature is 37C (98.6°F), pulse is 110/min, respirations are 14/min, and blood pressure is 160/100 mm Hg, Which of the following is the most appropriate next step in patient care? A) Admit the patient to the hospital for cocaine detoxification B) Admit the patient to the hospital for possible myocardial ischemia C) Educate the patient about the risks of cocaine use and send the patient home D) Transfer the patient to a psychiatric facility

B) Admit the patient to the hospital for possible myocardial ischemia

A 68-year-old man comes to the physician because of a 1-month history of light-headedness and tightness in his chest with exertion. He adds that the pain is worse after arguing with his wife, and the symptoms resolve with rest, He has a past history of lower gastrointestinal bleeding evaluation at that time was negative on upper endoscopy and colonoscopy. His temperature is 37°C (98.6°F), pulse is 85/min, respirations are 15/min, and blood pressure is 110175 mm Hg. Physical examination shows no abnormalities. His hemoglobin concentration is 8.2 g/dL, and hematocrit is 24%. Test of the stool for occult blood is positive. An ECG shows no abnormalities. Repeat colonoscopy shows no abnormalities. Which of the following is the most likely cause of this patient's gastrointestinal symptoms? A) Adenocarcinoma of the colon B) Angiodysplasia C) Diverticulitis D) Peutz-Jeghers syndrome E) Ulcerative colitis

B) Angiodysplasia

A 23 year old woman who is 6 months pregnant develops deep venous thrombosis. Intravenous administration of heparin is started. Seven days later, platelet count is 44,000/mm3. Which of the following is most likely responsible for the decreased platelet count in this patient? A) Antimegakaryocte antibodies B) Antiplatelet antibodies C) Heparin-induced hemodilution D) Hypersplenism E) Stem cell antibodies

B) Antiplatelet antibodies

A 38-year-old man comes to the physician because of a 1-week history of shortness of breath with exertion. His respirations are 12/min. Light palpation of the carotid artery shows the upstroke to be abnormally brisk and the downstroke to fall precipitously. Which of the following is the most likely cause of these physical examination findings? A) Aortic coarctation B) Aortic regurgitation C) Aortic stenosis D) Mitral regurgitation E) Mitral stenosis F) Ventricular septal defect

B) Aortic regurgitation

A 62-year-old man dies suddenly while playing tennis. He had no known cardiac risk factors and no history of coronary artery disease. At autopsy, examination shows a cardiac valve defect and concentric left ventricular hypertrophy. Which of the following valve abnormalities is most likely involved in his sudden death? A) Aortic insufficiency B) Aortic stenosis C) Mitral insufficiency D) Mitral stenosis E) Pulmonic insufficiency F) Pulmonic stenosis

B) Aortic stenosis

33. A 69-year-old man with benign prostatic hyperplasia develops urinary retention 1 day after undergoing removal of a melanoma, After catheterization, a drug administered that increases phosphoinositide turnover in the smooth muscle cells of the bladder fundus but has no effect on nicotinic synapses, This drug is most likely which of the following? A) Atropine B) Bethanechol C) Neostigmine D) Succinylcholine E) Trimethaphan

B) Bethanechol

A previously healthy 52-year-old man is brought to the physician because of a 2-month history of progressive difficulty swallowing, double vision, and slurred speech, His vital signs are within normal limits, Physical examination shows ptosis of the eyelids and a waddling gait. Muscle strength testing shows fatigable weakness of the neck, arms, hands, and fingers, Repetitive nerve stimulation shows a 25% decrease in muscle action potentials in several muscles. Which of the following mechanisms is the most likely cause of this patient's disease? A) Binding of acetylcholine by an antibody B) Binding of antibody to the acetylcholine receptor C) Decreased acetylcholine production D) Decreased release of acetylcholine from the receptor E) Decreased reuptake of acetylcholine

B) Binding of antibody to the acetylcholine receptor

45. A 4-year-old boy with asthma is brought to the emergency department because of a 4-hour history of difficulty breathing. His symptoms do not improve with the use of an inhaled ß-adrenergic agonist. Physical examination shows tachypnea and intercostal and subcostal retractions. Expiratory wheezing is heard bilaterally on auscultation. Following the administration of intravenous methylprednisolone, there is marked clinical improvement. Which of the following molecular pathways of methylprednisolone best explains this improvement? A) Binding to its cell-surface receptor and induction of phosphorylation of JAK2 and the STAT family B) Binding to its cytoplasmic receptor, translocation to the nucleus, and activation of transcription of target genes C) Binding to its G protein-coupled membrane receptor and stimulation of adenylyl cyclase D) Binding to its serine kinase receptor on the cell membrane that signals through SMADs E) Binding to its tyrosine kinase receptor on the cell membrane that undergoes autophosphorylation and activation of multiple kinases

B) Binding to its cytoplasmic receptor, translocation to the nucleus, and activation of transcription of target genes

49. A 60-year-old woman develops a secreting adenoma of the parathyroid gland, Which of the following is most likely to be decreased? A) Calcium absorption from the gut B) Calcium concentration in feces C) Calcium concentration in muscle D) Calcium concentration in plasma E) Calcium loss from bone

B) Calcium concentration in feces

A 37-year-old woman is brought to the emergency department after her husband found her unconscious. Her temperature is 36°C (96.8°F), pulse is 128/min. and blood pressure is 70/40 mm Hg. Physical examination shows cool, pale extremities, jugular venous distention, faint peripheral pulses, and crackles over the bottom two thirds of both lung fields, Heart sounds are normal, and there are no murmurs, She withdraws to painful stimuli in el! four extremities, This patient is most likely experiencing which of the following types of shock? A) Anaphylactic B) Cardiogenic C) Hypovolemic D) Neurogenic E) Septic

B) Cardiogenic

24. A new compound is taken up by bacterial cells. No energy is necessary for uptake, and the compound is not concentrated in the cell, Which of the following best describes this mechanism of transport? A) ATP-dependent active transport B) Carrier-mediated diffusion C) Group translocation D) Phosphorylation-linked transport E) Proton-gradient-mediated transport

B) Carrier-mediated diffusion

A 67-year-old man comes to the physician because of a 2-month history of unexplained weight loss. He has a 50-year history of type 1 diabetes mellitus. He underwent gallstone removal 12 years ago. He has smoked 1 pack of cigarettes daily for 45 years. He is 188 cm (6 ft 2 in) tall and weighs 120 kg (265 lb): BMI is 34 kg/m2. Physical examination shows no other abnormalities. Serum studies show a calcium concentration of 11 mg/cIL. An abdominal CT scan shows a pancreatic mass. A biopsy specimen of the mass shows pancreatic adenocarcinoma. Which of the following is the strongest predisposing risk factor for pancreatic cancer in this patient? A) BMI of 34 kg/m2 B) Cigarette smoking C) History of gallstone disease D) Hypercalcemia E) Type 1 diabetes mellitus

B) Cigarette smoking

1. A 21-year-old woman at 30 weeks' gestation is admitted to the hospital because of contractions for 1 hour, Fetal ultrasonography shows that the ductus arteriosus has closed, Inhibition of which of the following enzymes is the most likely cause of this finding? A) Adenylyl cyclase B) Cyclooxygenase C) Glutathione S-transferase D) Guanylyl cyclase E) 5-Lipoxygenase

B) Cyclooxygenase

45. A 1400-g (3-lb 2-oz) female newborn is delivered to a 33-year-old primigravid woman at 37 weeks' gestation. Apgar scores are 6 and 8 at 1 and 5 minutes, respectively. Cardiac examination shows a rough, grade 2/6, continuous. machine-like murmur along the left sternal border. as well as increased precordial activity and bounding peripheral pulses. The most appropriate treatment will inhibit the activity of which of the following enzymes in this patient? A) Angiotensin-converting enzyme B) Cyclooxygenase C) Guanylyl cyclase D) Nitric oxide synthase E) Phospholipase A2

B) Cyclooxygenase

34. A 32-year-old woman has had fecal incontinence since giving birth to her first child 1 year ago. Which of the following is the most likely cause of the fecal incontinence in this patient? A) Achalasia of the internal anal sphincter B) Damage to the anal sphincter C) Increased pressure of the anal sphincter D) Spastic contraction of the puborectalis muscle E) Spastic external anal sphincter

B) Damage to the anal sphincter

A 41 year old woman at 34 weeks' gestation undergoes an amniocentesis because of uncontrolled hypertension. Amniotic fluid shows a decreased lecithin-to-sphingomyelin ratio of 1.5:1 (N=3.1). If the newborn is born at this time, which of the following pulmonary findings is most likely to develop in the newborn during the first 48 hours after delivery? A) Decreased airway resistance B) Decreased functional residual capacity C) Decreased respiratory rate D) Increased static lung compliance E) Increased total lung capacity

B) Decreased functional residual capacity

A 43-year-old man with a 10-year history of alcoholism comes to the physician because of a change in skin color. The patient says that he has been taking two extra-strength acetaminophen tablets every 4 to 6 hours for the past 3 days for a severe headache. Physical examination shows jaundice. Laboratory studies show an increased prothrombin time and a markedly increased serum AST activity. Alteration in which of the following metabolites within hepatocytes is most likely associated with his illness? A) Decreased glucuronide conjugates B) Decreased glutathione C) Decreased NAD+ D) Decreased NADH E) Increased glucuronide conjugates F) Increased glutathione G) Increased NAD+ H) Increased NADH

B) Decreased glutathione

28. A 60-year-old man with a 20-year history of hypertension comes to the physician because of shortness of breath and fatigue for 3 months. The shortness of breath is exacerbated by climbing stairs. His pulse is 80/min, and blood pressure is 100/70 mm Hg. Physical examination shows peripheral cyanosis. Crackles are heard bilaterally on auscultation of the chest. Cardiac examination shows an S3. There is 2+ pitting ankle edema. Which of the following is the most likely set of cardiac output and central venous pressure (CVP) findings in this patient? Cardiac Output CVP A) Decreased decreased B) Decreased increased C) Decreased no change D) Increased decreased E) Increased increased F) Increased no change G) No change decreased H) No change increased I) No change no change

B) Decreased increased

A 43-year-old woman comes to the physician because of progressive difficulty walking during the past 3 months. Neurologic examination shows weakness and decreased muscle bulk of the lower extremities. Patellar and Achilles tendon reflexes are diminished. Sensations of joint position, pain, and temperature in the lower extremities are normal. Which of the following is the most likely cause of the findings in this patient? A) Acute peripheral neuropathy B) Degeneration of motoneurons of the lumbar cord C) Demyelination of the corticospinal pathways D) Loss of afferent la axons innervating muscle spindles E) Myotonic muscular dystrophy

B) Degeneration of motoneurons of the lumbar cord

A 1045-g (2-lb 5-oz) male newborn delivered at 28 weeks' gestation develops rapid breathing, grunting, and subcostal retractions shortly after birth. Coarse rhonchi are heard on auscultation of the chest. A chest x-ray shows fine densities with a reticulogranular appearance bilaterally. Arterial blood gas analysis on room air shows decreased pH and Po2 and increased Pco2. A deficiency of which of the following is the most likely cause of the pulmonary disorder in this patient? A) Diacylglycerol B) Dipalmitoylphosphatidylcholine C) Phosphatidylserine D) Sphingomyelin E) Surfactant protein D

B) Dipalmitoylphosphatidylcholine

A 16-year-old girl comes to the physician because of a 2-year history of intermittent fainting that occurs while she is standing: these episodes have increased in frequency during the past 6 months. Her blood pressure is 110/80 mm Hg while supine and 60/40 mm Hg while standing. Neurologic examination shows no other abnormalities. Plasma studies show an undetectable norepinephrine concentration and a marked increase in dopamine concentration when rising from a supine position to standing. This patient most likely has a deficiency of which of the following? A) Amino acid decarboxylase B) Dopamine ß-hydroxylase C) Monoamine oxidase D) Norepinephrine transporter E) Tyrosine hydroxylase

B) Dopamine ß-hydroxylase

A 16-year-old girl is brought to the emergency department by ambulance 20 minutes after a generalized tonic-clonic seizure at borne, She has two more seizures after arrival. She is unable to urinate. She appears confused. Her pulse is 120/min, respirations are 24/min, and blood pressure is 170/109 mm Hg. The pupils are equal and reactive. Abdominal examination shows an enlarged uterus, Cranial nerves are intact. Deep tendon reflexes are 4+ with 1+ clonus bilaterally. Laboratory studies show: Hematocrit 42%, Leukocyte count 15,000/mm3, Platelet count 90,000/mm3, Serum: Creatinine 1.7 mg/dL, AST 96 U/L, ALT 79 U/L, Lactate dehydrogenase 328 U/L, Uric acid 9.6 mg/dL Ultrasonography shows a normal fetus consistent in size with a 30-week gestation. Which of the following is the most likely diagnosis? A) Drug overdose B) Eclampsia C) Epilepsy D) Renal disease E) Sepsis

B) Eclampsia

A female newborn is delivered at term to a 35-year-old primigravid woman. Pregnancy was complicated by untreated maternal Graves disease. Her respirations are 66/min. Physical examination shows strider, nasal flaring, intercostal retractions, and an asymmetric neck mass. Which of the following is the most likely cause of the newborn's stridor? A) Cystic hygroma B) Enlarged thyroid gland C) Thyroglossal duct cyst D) Tracheomalacia E) Vascular ring

B) Enlarged thyroid gland

A 30-year-old nulligravid woman comes to the physician because she has been unable to conceive for 2 years. She has had a 36-kg(80-lb) weight gain during this period. Her last menstrual period was 4 months ago. Menses had occurred at regular 28-day intervals until the age of 25 years. They now occur every 90 to 120 days. She is 168 cm (5 ft 6 in) tall and now weighs 98 kg (215 lb); BMI is 35 kg/m2. Her blood pressure is 130/90 mm Hg. Physical and pelvic examinations show no other abnormalities. A pregnancy test result is negative. Serum studies show: Prolactin 15 ng/mL, Thyroid-stimulating hormone 2 µU/mL,Follicle-stimulating hormone 10 mlu/mL, Luteinizing hormone 28 mlu/mL. The patient has withdrawal bleeding after a progestin challenge test. The most likely cause of this patient's oligomenorrhea is an increased production of which of the following hormones? A) Androgen in the adrenal glands B) Estrogen in adipose tissue C) Gonadotropin in the pituitary gland D) Gonadotropin-releasing hormone in the hypothalamus E) Progesterone in the ovaries

B) Estrogen in adipose tissue

38. A 25-year-old man develops a temperature of 38.6°C (101.5°F) after running his first 10-kilometer (6.2-mile) race. This man's temperature is most likely to return to normal because of which of the following mechanisms? A) Central arteriolar vasodilation B) Evaporation of sweat C) Increased minute ventilation D) Increased muscle tone E) Peripheral arteriolar vasoconstriction F) Peripheral venous vasoconstriction

B) Evaporation of sweat

A 3-year-old boy and his 5-year-old brother bruise easily and have recurrent hemarthroses. Both parents are healthy, but the mother has two younger brothers with the same symptoms and a maternal uncle who died at 8 years of age after mild head trauma, Partial thromboplastin time is prolonged. The most likely defect involves which of the following? A) Factor V (proaccelerin) B) Factor VIII (antihemophilic factor) C) Factor XIII (transglutaminase) D) Thrombin E) von Willebrand factor

B) Factor VIII (antihemophilic factor)

A 60-year-old man comes to the physician because of weakness and light-headedness for 3 months. His diet consists of a healthy balance of fruits, vegetables, whole grains, and lean chicken and fish, He has a 10-year history of chronic autoimmune (Hashimoto) thyroiditis treated with thyroxine. Physical examination shows pallor. Serum studies show increased concentrations of methylmalonic acid and total homocysteine. A peripheral blood smear shows megaloblastic anemia. Which of the following is the most likely cause of the findings in this patient? A) Cystathionine ß-synthase deficiency B) Failure of intrinsic factor production C) Folic acid deficiency D) Helicobarter pylori gastritis E) Methylmalonyl-CoA mutase deficiency

B) Failure of intrinsic factor production

A 62-year-old man develops a pericardial friction rub 3 days after an acute myocardial infarction_ Which of the following is the most likely cause of the friction rub? A) Cardiac tamponade B) Fibrinous pericarditis C) Left ventricular failure D) Mitral valve prolapse E) Purulent pericardial effusion

B) Fibrinous pericarditis

A 28-year-old woman at 18 weeks' gestation has palpitations. Laboratory studies show an increased serum total thyroxine (T4) concentration, Which of the following tests is best to confirm a diagnosis of hyperthyroidism in this patient? A) Fine-needle aspiration biopsy of the thyroid gland B) Free T4 C) Radioactive iodine uptake D) Serum total triiodothyronine (T3) E) Thyroid antibodies

B) Free T4

A 20-year-old man has had hemoptysis, hematuria, and a 10-kg (22-lb) weight loss over the past 3 months. Laboratory studies show a hypochromic microcytic anemia. Which of the following sets of microscopic findings is most likely in tissue obtained on biopsy of his kidney? Light Microscopy; Deposition in Glomerular Basement Membrane; Electron Microscopy A) Diffusely hypercellular glomeruli; granular lgG and C3; subepithelial humps B) Glomerular crescents; linear lgG and C3; no deposits C) Normal glomerular cellularity; granular IgG and C3; subepithelial deposits D) Normal glomerular cellularity;negative; loss of foot processes

B) Glomerular crescents; linear lgG and C3; no deposits

27. A 6-year-old girl is brought to the physician for a follow-up examination. She has been receiving treatment with potassium citrate monohydrate since the diagnosis of renal tubular acidosis was made at the age of 2 years. Physical examination shows no abnormalities, Serum studies show: Na+ 142 mEq/L, K+ 3.5 mEq/L, Cl- 115 mEq/L, HCO3- 18 mEq/L, Urea nitrogen 9 mg/dL, Creatinine 0.9 mg/dL A defect in renal ammoniagenesis is suspected. Which of the following substrates is the most likely source of ammonia production in this patient? A) Creatinine B) Glutamine C) Glycine D) Leucine E) Urea

B) Glutamine

35. A 51-year-old man with a 5-year history of Wegener granulomatosis comes to the physician for a follow-up examination. He has general malaise, fatigue, myalgia, and arthralgia. Which of the following additional findings is most likely to indicate exacerbation of this patient's disease? A) Glucosuria B) Hematuria C) Hypokalemia D) Hyponatremia E) Increased glomerular filtration rate

B) Hematuria

A 52 year old man is brought to the emergency department 30 minutes after the sudden onset of substernal chest pain. The pain has persisted despite self administration of three doses of sublingual nitroglycerin. His pulse is 90/min and blood pressure is 114/70 mm Hg. Physical examination shows diaphoresis. The lungs are clear to auscultation. Cardiac examination shows an S4. An ECG shows ST-segment elevation in the inferior leads. Intravenous morphine is administered and he develops generalized pruritus and warmth 1 minute later. His pulse now is 120/min and blood pressure is 90/50 mm Hg. Physical examination shows facial flushing. Which of the following substances is the most likely cause of the new symptoms and findings in this patient? A) Epinephrine B) Histamine C) Nitric oxide D) Serotonin E) Thromboxane A2

B) Histamine

An 80-year-old woman comes to the emergency department because of a 2-day history of "feeling funny." She says, "I have lost my pep," She has never been admitted to the hospital, but she has a history of poorly controlled hypertension, She just started a medication 2 weeks ago but does not recall the name. Her blood pressure is 130/85 mm Hg. Physical examination shows no abnormalities. Her serum potassium concentration is 3 mEq/L, Which of the following drugs is the most likely cause of this patient's condition? A) Clonidine B) Hydrochlorothiazide C) Lisinopril D) Metoprolol E) Triamterene

B) Hydrochlorothiazide

A 24-year-old woman with Marian syndrome comes to the physician 1 week after a home pregnancy test result was positive. Previous evaluations showed mild coarctation of the thoracic aorta with mild dilation of the ascending aorta, and mild aortic regurgitation, but no significant heart failure, If her pregnancy were to proceed, which of the following mechanisms would most likely increase the amount of aortic regurgitation in this patient? A) Decreased vascular resistance B) Hypervolemia C) Increased cutaneous blood flow D) Increased metabolic rate E) Weight gain

B) Hypervolemia

A previously healthy 54-year-old man comes to the clinic because of light-headedness for 6 hours. His symptom began after skiing at a resort at an altitude of 2743 m (9000 ft). He has been taking a carbonic anhydrase inhibitor since 2 days before arrival at the resort, His blood pressure is 110/60 mm Hg while sitting and 95/50 mm Hg while standing. Physical examination shows no other abnormalities, Which of the following is the most likely cause of his orthostatic hypotension? A) High-altitude sickness B) Hypovolemia C) Hypoxia D) Impaired sympathetic nerve activity E) Respiratory alkalosis

B) Hypovolemia

A 50-year-old man who has smoked 2 packs of cigarettes a day for 34 years has experienced gradually increasing shortness of breath on exertion, chronic cough productive of thick sputum, and frequent episodes of wheezing. The anteroposterior diameter of his chest is increased. Diminished breath sounds and scattered rhonchi are audible throughout the lung fields. Which of the following laboratory abnormalities is expected? A) Decreased blood Pco2 B) Increased blood HCO3- C) Increased blood pH D) Increased urinary excretion of HCO3- E) Increased urinary pH

B) Increased blood HCO3-

14. A 53-year-old man with angina pectoris is treated with nitroglycerin. Which of the following actions of nitroglycerin is most likely to be responsible for symptomatic improvement? A) Impaired transmembrane calcium flux B) Increased concentration of cyclic GMP in smooth muscle C) Induction of the release of acetylcholine by endothelial cells D) Induction of the release of interleukin-1 (IL-1) by endothelial cells E) Stimulation of autonomic nervous system tone

B) Increased concentration of cyclic GMP in smooth muscle

A 2-month-old male infant with a ventricular septal defect is brought to the physician because of poor weight gain and a 5-day history of rapid breathing and difficulty feeding. The diagnosis of congestive heart failure is made, and an operation to place a synthetic patch across the septal defect is planned. Compared with heart pressures prior to the operation, which of the following heart pressures are most likely 1 month after a successful operation? Left Ventricular Pressure Right Ventricular Pressure Left Atrial Pressure A) Increased increased no change B) Increased decreased decreased C) No change increased decreased D) Decreased no change decreased E) Decreased decreased increased

B) Increased decreased decreased

A 50-year-old man comes to the physician because of a 2-month history of pain of his wrists, changes in skin color, and progressive fatigue. His brother has type 2 diabetes mellitus and cirrhosis. Physical examination shows bronze-colored skin, tenderness of the metacarpophalangeal joints in both hands. and hepatosplenomegaly. Serum studies show: AST: 100 U/L, ALT: 110 U/L, Ferritin: 1200 ng/mL, Total iron-binding capacity: 200 µg/dL (N=250-400), Transferrin saturation: 80% (N=20-50) Analysis of a liver biopsy specimen shows a markedly increased iron concentration and cirrhosis. Which of the following is the most likely cause of the findings in this patient? A) Increased erythropoietin action B) Increased intestinal iron absorption C) Increased oral iron intake D) Decreased erythropoiesis E) Decreased iron excretion F) Decreased serum transferrin concentration

B) Increased intestinal iron absorption

A 30 year old woman comes to the physician for a health maintenance examination. She is training for a marathon and has been running up to 20 miles daily. She states that as long as she has adequate caloric intake, she feels well on long-distance runs of up to 20 miles. Physical examination shows no abnormalities. Her fasting serum glucose concentration is 60 mg/dL. After her glucose stores have been depleted, which of the following organs, in addition to the liver, is most likely to release newly produced glucose in this patient? A) Adrenal glands B) Kidney C) Pancreas D) Stomach E) Thyroid gland

B) Kidney

A 45 year old man is brought to the emergency department after being lost outdoors in the Rocky Mountains for 10 days. He has had a 5 kg (11-lb) weight loss during this period. His serum glucose concentration is 92 mg/dL. A serum glucose concentration within the reference range was most likely maintained in this patient via glucose synthesis in the liver and in which of the following organs? A) Adrenal glands B) Kidney C) Large intestine D) Pancreas E) Small intestine F) Spleen

B) Kidney

A 65-year-old woman with hypertension comes to the emergency department because of a 3-hour history of headache, chest pain, and shortness of breath. Current medications are aspirin, furosemide, and lisinopril_ Her respirations are 20/min, and blood pressure is 230/110 mm Hg_ The physician plans to initiate treatment with a nonselective antagonist that blocks α1 and ß-adrenoreceptors, Which of the following drugs is most appropriate? A) Hydrarazine B) Labetalol C) Phenoxybenzamine D) Phentolamine E) Trimethaphan

B) Labetalol

A pharmaceutical company is trying to develop a long-acting weight-loss agent that mimics the activity of a naturally occurring peptide that originates in adipose tissue, signals the brain about the amount of stored fat, and suppresses appetite by its action in the central nervous system. Their best strategy for identifying useful peptides would be to assess the binding of candidate peptides to the receptor for which of the following chemical mediators? A) Cholecystokinin B) Leptin C) Neurotensin Y D) Seratonin E) Somatostatin F) Substance P

B) Leptin

A 5-year-old boy is brought to the physician because of an 8-month history of recurrent middle ear and upper respiratory tract infections. He has a history of impaired motor and cognitive function. Physical examination shows hepatosplenomegaly. Analysis of cultured skin fibroblasts shows the presence of inclusion bodies and a deficiency in the activity of N-acetylglucosamine-1-phosphotransferase. The cells secrete abnormally large amounts of acid hydrolases into the culture medium. These enzymes are unable to target which of the following organelles in this patient? A) Endoplasmic reticulum B) Lysosomes C) Mitochondria D) Nucleus E) Peroxisomes

B) Lysosomes

7. A 1-week-old girl is brought to the physician for a follow-up examination after newborn screening showed a possible defect in fatty acid oxidation. Physical examination shows no abnormalities. Which of the following is the most appropriate next step in diagnosis? A) Arterial blood gas analysis B) Measurement of serum acylcarnitine concentrations C) Measurement of serum amino acid concentrations D) Measurement of serum electrolyte concentrations E) Measurement of serum lactic acid concentration

B) Measurement of serum acylcarnitine concentrations

12. A 73-year-old man is brought to the physician by his wife because of a change in behavior, decreased appetite. and intermittent constipation during the past 4 months. He also has had a 4-kg (10-lb) weight gain during this period. His wife says, 'He takes forever to get out of bed each day. and he talks so slowly these days. He is always 'forgetting things like where he put his keys." The patient says, "1 don't think I have a memory problem. but I feel apathetic all the time." Physical examination shows an enlarged tongue. He has slowed speech and difficulty following complex commands. He recalls only one of three objects after 5 minutes. Which of the following is most likely to establish the diagnosis? A) Measurement of fasting serum glucose concentration B) Measurement of serum thyroid-stimulating hormone concentration C) Serologic testing for syphilis D) EEG E) Lumbar puncture

B) Measurement of serum thyroid-stimulating hormone concentration

An otherwise healthy 35-year-old man comes to the physician because of several episodes of dizziness and fainting during the past 2 months, His father and several paternal uncles died suddenly. Physical examination shows no abnormalities. Serum lipid studies are within the reference range. Angiography shows no blockage of the coronary arteries. An ECG shows a prolonged OT interval. A decreased activity of which of the following is the most likely cause of these findings? A) Inward activating potassium channel B) Outward (delayed) rectifying potassium channel C) Voltage-gated fast sodium channel D) Voltage-gated L-type calcium channel E) Voltage-gated T-type calcium channel

B) Outward (delayed) rectifying potassium channel

43. A 48-year-old woman conies to the physician because of a 3-month history of fatigue: weakness, loss of appetite, and weight loss. Her serum parathyroid hormone concentrations are increased. X-rays of the skeletal system show generalized osteopenia, with subperiosteal resorption of bone within the phalanges. Which of the following mechanisms is the most likely cause of the skeletal changes observed in this patient? A) Impaired calcification of preformed osteoid B) Paracrine stimulation of osteoclasts by osteoblasts C) Parathyroid hormone-mediated induction of calcitonin D} Parathyroid hormone-mediated induction of interleukin-1 (IL-1) (1-1) E) Parathyroid hormone-mediated production of transforming growth factor-ß1

B) Paracrine stimulation of osteoclasts by osteoblasts

A 55-year-old woman comes to the physician for a routine health maintenance examination. Her last menstrual period was 8 years ago. She takes no medications. Physical examination shows no abnormalities. The physician orders fasting serum lipid studies to screen for factors likely to predispose her to cardiovascular disease. Compared with a healthy 25-year-old woman, which of the following is most likely to be decreased in this patient? A) Free fatty acids B) Serum HDL-cholesterol concentration C) Serum IDL-cholesterol concentration D) Serum LDL-cholesterol concentration E) Serum triglyceride concentrations

B) Serum HDL-cholesterol concentration

A 34-year-old man is evaluated after becoming lightheaded after running 12 miles of a marathon on a hot day. His pulse is 130/min and his blood pressure is 80/60 mm Hg. Which of the following changes in his autonomic nervous system most likely occurred? Sympathetic Efferent Activity Parasympathetic Efferent Activity A) UP UP B) UP DOWN C) <-> UP D) <-> DOWN E) DOWN UP F) DOWN DOWN

B) UP DOWN

A 68-year-old man comes to the physician because of a 10-month history of shortness of breath and swelling of his feet. He has a family history of cardiovascular disease, He has smoked 2 packs of cigarettes daily for 50 years, His pulse is 80/min, respirations are 241min, and blood pressure is 150/80 mm Hg. Physical examination shows 3+ pitting edema of the lower extremities. Diffuse, scattered wheezes are heard bilaterally on auscultation of the chest. Cardiac examination shows a grade 2/6 pansystolic murmur heard best at the lower left sternal border, which increases on inspiration. The point of maximal impulse is palpated in the sub-xiphoid area S1 and S2 sounds are distant. Abdominal examination shows a liver span of 14 cm. Which of the following is the most likely diagnosis? A) Aortic stenosis B) Cardiac amyloidosis C) Cor pulmonale D) Coronary artery disease E) Primary pulmonary hypertension

C) Cor pulmonale

A 40-year-old African American woman comes to the physician because of a 2-week history of fever, malaise, and dyspnea. Her temperature is 36.7°C (98°F), and respirations are 20/min. Physical examination shows erythema nodosum, parotid enlargement, and hepatosplenomegaly. Her serum calcium concentration is 16 mg/dL A CT scan of the chest shows bilateral hilar adenopathy, Serum studies are most likely to show an increased concentration of which of the following? A) Calcitonin B) Cholecalciferol C) 1,25-Dihydroxycholecalciferol D) 24,25-Dihyclroxycholecalciferol E) Parathyroid hormone

C) 1,25-Dihydroxycholecalciferol

A 25-year-old woman comes to the physician because of a 3-year history of irregular menses, Menarche was at the age of 14 years. Puberty began at the age of 12 years and progressed normally. Her blood pressure is 116/62 mm Hg. Physical examination shows increased hair growth on the face and chest. Pelvic examination shows clitoromegaly and a normal-appearing uterus. Serum studies show increased concentrations of 17-hydroxyprogesterone and androstenedione. This patient most likely has a deficiency of which of the following enzyme activities? A) 11ß-Hydroxylase B) 17α-Hydroxylase C) 21-Hydroxylase D) 3ß-Hydroxysteroid dehydrogenase E) 5α-Reductase

C) 21-Hydroxylase

5. During a clinical study of calcium and phosphorus metabolism, a 50-year-old man undergoes a series of laboratory studies. His serum calcium, phosphorus, and parathyroid hormone concentrations are within the reference ranges. He is given an infusion of 2 g of calcium chloride over 2 hours. His serum calcium concentration now is 11.5 mg/dL. Compared with the pre-infusion level, the serum concentration of which of the following substances is most likely to be increased at this time? A) 7-Dehydrocholesterol B) 1,25-Dihydroxycholecalciferol C) 24,25-Dihydrioxycholecalciferol D) Previtamin D3 E) Vitamin D3

C) 24,25-Dihydrioxycholecalciferol

14. An 18-year-old woman is brought to the emergency department 12 hours after she ingested approximately 100 aspirin tablets in a suicide attempt. Physical examination shows tachypnea. Which of the following sets of laboratory findings would most likely be found in this patient? pH Pco2(mm Hg) HCO3-(mEq/L) A) 6.90 60 10 B) 7.30 60 29 C) 7.32 15 8 D) 7.44 60 40 E) 7.54 40 40 F) 7.64 20 40

C) 7.32 15 8

18. A 12-year-old boy with asthma is brought to the physician because of wheezing for 2 hours. Treatment with an inhaled bronchodilator has not completely resolved his symptoms. Which of the following portions of the airway is most susceptible to flow limitation due to smooth muscle contraction in this patient? A) Alveolar duct B) Alveolus C) Bronchiole D) Nasal passage E) Trachea

C) Bronchiole

3. A 52-year-old man has chronic pancreatitis. Deficiency of which of the following is the most likely cause of this patient's inability to digest triglycerides? A) Carbonic anhydrase B) Cholesterol ester hydrolase C) Colipase D) Monitor peptide E) Phospholipase A2

C) Colipase

26. A 52-year-old woman with breast cancer comes to the physician for a follow-up examination. She had a 4-week course of radiation treatment 6 months ago. Her respirations are 26/min. Physical examination shows no recurrence of the cancer A CT scan of the chest shows bilateral patches of atelectasis in the upper lung fields. The atelectasis in this patient most likely developed because of which of the following primary pathophysiologic processes? A) Compression B) Consolidation C) Contraction D) Obstruction E) Resorption

C) Contraction

A 35-year-old woman participates in a study of a new diagnostic test of steroid hormones in saliva. Menses occur at regular 28-day intervals, and she has been shown to ovulate on day 14_ When her saliva is tested on a particular day of the cycle, laboratory studies show an increased concentration of estradiol and decreased concentration of progesterone. On which of the following days in this patient's menstrual cycle were these hormones most likely measured? A) Day 1 B) Day 6 C) Day 12 D) Day 18 E) Day 26

C) Day 12

33. A 25-year-old woman with polycystic kidneys has a 3-month history of progressive weakness and fatigue, headaches, hypertension, loss of appetite, and itching. Her serum creatinine concentration is 4 mg/dL. Which of the following laboratory abnormalities in serum is expected in this patient? HCO3- Inorganic PO4 Parathyroid Hormone A) Decreased decreased increased B) Decreased decreased normal C) Decreased increased increased D) Increased increased decreased E) Increased normal decreased F) Normal decreased decreased

C) Decreased increased increased

41. A 66-year-old man comes to the physician because of a 2-month history of the inability to maintain an erection. He also has had fatigue and difficulty sleeping and concentrating during this period. Three months ago, he had a cerebral infarction with right hemiparesis that has gradually resolved. Physical examination shows no abnormalities. Which of the following additional pairs of findings in this patient is most likely on history taking? Libido Nocturnal Erections A) Normal normal B) Normal decreased C) Decreased normal D) Decreased decreased

C) Decreased normal

A 25-year-old man receives a bupivacaine injection for median nerve block as treatment of carpal tunnel syndrome, Which of the following is the mechanism of action of this drug on the membrane? A) Activation of the Na+—K+ ATPase B) Decreased permeability to Ca2+ C) Decreased permeability to Na+ D) Increased reuptake glutamate E) Inhibition of the Na+/Ca2+ antiporter

C) Decreased permeability to Na+

A 66-year-old man comes to the physician because of a 6-month history of decreased exercise tolerance and shortness of breath with exertion. His pulse is 92/min, respirations are 22/min, and blood pressure is 120/80 mm Hg, Auscultation of the chest shows bilateral basilar crackles. Cardiac examination shows an S3 gallop. A grade 2/6 holosystolic murmur is heard best at the apex. The apical impulse is palpated at the anterior axillary line at the sixth intercostal space. Echocardiography shows left and right ventricular enlargement, with a global decrease in systolic function, Which of the following is the most likely diagnosis? A) Aortic valve stenosis B) Coronary artery disease C) Dilated cardiomyopathy D) Hypertensive heart disease E) Pericardial tamponade

C) Dilated cardiomyopathy

A 22-year-old man who is a professional cyclist undergoes extensive physiologic testing as part of his training regimen. His resting pulse is 33/min, and blood pressure is 110/62 mm Hg. Echocardiography shows dilated ventricles with normal function and a left ventricular ejection fraction of 75%. Which of the following best describes the findings in this patient? A) Congestive cardiomyopathy B) Diastolic dysfunction C) Eccentric hypertrophy D) Hypertrophic cardiomyopathy E) Increased myocardial stiffness

C) Eccentric hypertrophy

31. A 50-year-old woman with chronic obstructive pulmonary disease comes to the physician because of a 3-month history of progressive shortness of breath. Physical examination shows jugular venous distention. A loud pulmonary component of S2 is heard on auscultation. Pulmonary function tests show an FEV1:FVC ratio of 20% and a severely decreased diffusing capacity for carbon monoxide. Based on these findings, which of the following is most likely decreased in this patient's pulmonary vascular smooth muscle? A) Adventitial collagen matrix deposition B) Cytosolic phospholipase Al activity C) Endothelial nitric oxide synthase production D) Endothelin expression E) Voltage-gated calcium channel a-subunit production

C) Endothelial nitric oxide synthase production

3. A 45-year-old woman comes to the physician because she recently was diagnosed with cholelithiasis, She asks the physician why her abdominal pain is intermittent and not constant. The physician explains that pain is produced when the gallbladder contracts against the gallstone that is obstructing the cystic duct Secretion of hormones from which of the following cells is the most Likely cause of this contraction? A) Centroacinar cells of the exocrine pancreas B) Chief cells of the gastric mucosa C) Enteroendocrine cells of the small intestine D) Gastric parietal cells E) Vasoactive intestinal polypeptide-producing cells of the jejunal mucosa

C) Enteroendocrine cells of the small intestine

A 15-year-old boy is brought to the physician for an examination prior to participating on a sports team. He is 191 cm (6 ft 3 in) tall and weighs 75 kg (165 lb); BMI is 21 kg/m2. His pulse is 85/min and bounding, and blood pressure is 110/40 mm Hg. Ophthalmologic examination shows the dislocation of the right lens. Cardiac examination shows a pectus excavatum. A grade 3/6, decrescendo diastolic murmur is heard at the aortic area, Chest x-rays show a wide mediastinum. This patient most likely has a genetic defect in the production of which of the following? A) Collagen, type II B) Elastin C) Fibrillin D) Hyaluronate E) Proteoglycan

C) Fibrillin

A 44-year-old man with a 1-year history of angina pectoris conies to the emergency department because of increasingly severe chest pain during the past 2 days. He has had five previous similar episodes, which required treatment with increasing doses of nitroglycerin to resolve, His temperature is 37°C (98.6cF), pulse is 105 /min, respirations are 16/min, and blood pressure is 150/90 mm Hg. Cardiac examination shows an S4, An EGG shows ST-segment depression in the precordial leads. In addition to aspirin, heparin, and nitroglycerin, he is given a dose of a monoclonal antibody against the platelet llb/lla receptor. This antibody will most likely prevent binding of which of the following substances to platelets? A) Adenosine B) ADP C) Fibrinogen D) Serotonin E) Thrombin F) Thromboxane A2

C) Fibrinogen

A 29-year-old woman comes to the physician for advice on losing weight. She has been taking thyroxine for several months in an attempt to lose weight. She obtained the thyroxine from a friend who had a prescription. Her thyroid function is normal. Which of the following findings is most likely on histopathologic examination of the thyroid gland? A) C cell hypoplasia B) Columnar metaplasia C) Follicular atrophy D) Glandular hypertrophy E) Squamous metaplasia

C) Follicular atrophy

A 5-year-old boy is stung on his foot by a bee. Within 30 minutes the local area is edematous. The extravascular accumulation of fluid is most directly related to which of the following? A) Demargination of leukocytes B) Fibrin thrombi C) Gap formation between endothelial cells D) Vasoconstriction E) Vasodilation

C) Gap formation between endothelial cells

22. In a healthy person, the thyroid gland produces and secretes thyroxine (T4), maintaining metabolism, Which of the following best describes the T4-binding substance in the blood and the type of cellular receptor for T4 in the peripheral tissues? T4-binding Substance in the Blood Receptor That Interacts With T4 in the Peripheral Tissues A) Globulin cell membrane receptor B) Globulin mitochondrial C) Globulin nuclear D) Lipid cell membrane receptor E) Lipid mitochondrial F) Lipid nuclear

C) Globulin nuclear

An investigator breeds a transgenic strain of mice that develops severe fasting hypoglycemia. Administration of glucagon does not correct the hypoglycemia, but administration of epinephrine results in an increase in the serum glucose concentration, Which of the following proteins is most likely defective in this mouse strain? A) Adenylyl cyclase B) Glucagon receptor C) Glycogen phosphorylase D) Heterotrimeric G protein E) Phosphorylase kinase F) Protein kinase A

C) Glycogen phosphorylase

A 17-year-old boy is brought to the physician by his mother because she is concerned about his physical development She says that he is not developing the way his two older brothers did, He has no history of major medical illness. He is at the 10th percentile for height and 25th percentile for weight. Serum studies show a follicle-stimulating hormone concentration of 5 mIU/mL and testosterone concentration of 8 nmol/L (N=10-35). Which of the following physical findings is most likely in this patient? A) Enlarged testes B) Excess body hair C) Gynecomastia D) Low-pitched voice E) Strong muscular development

C) Gynecomastia

A 15-year-old girl is brought to the emergency department 12 hours after she ingested an entire bottle (100 capsules) of vitamin D in a suicide attempt. The patient says that she is not in any pain. Physical examination shows no abnormalities. Laboratory studies are within the reference ranges. At a follow-up visit 1 month later, her serum calcium concentration is 10.4 mg/dL. Which of the following mechanisms is the most likely cause of the increased serum calcium concentration in this patient? A) Decreased excretion of calcium from the gastrointestinal tract B) Decreased osteoclast activity in bone C) Increased absorption of calcium in the gastrointestinal tract D) Increased 1-hydroxylase activity in the kidney E) Increased osteoblast activity in bone

C) Increased absorption of calcium in the gastrointestinal tract

A 75-year-old man comes to the physician because of a 2-year history of decreased force of his urinary stream and a need to urinate several times throughout the night. His serum urea nitrogen (BUN) concentration is 55 mg/dL, and serum creatinine concentration is 5.0 mg/dL, Ultrasonopraphy of the urinary tract shows bilateral hydronephrosis and dilated ureters. Which of the following is the most likely mechanism of this patient's renal failure? A) Decreased hydrostatic pressure in glomerular capillary B) Decreased renal plasma flow C) Increased hydrostatic pressure in Bowman space D) Precipitation of proteins in the renal tubules E) Precipitation of uric acid in the renal tubules

C) Increased hydrostatic pressure in Bowman space

22. A 21-year-old man loses 15% of his total blood volume within 2 minutes after a motor vehicle collision, Which of the following findings is most likely? A) Chemoreceptor-mediated vasoconstriction B) Decreased sympathetic nerve activity to the veins C) Increased sympathetic nerve traffic to the sinoatrial node D) Increased tonic release of atrial natriuretic peptide from the atria E) Increased vagal efferent nerve activity to the sinoatrial node

C) Increased sympathetic nerve traffic to the sinoatrial node

A 52 year old man is brought to the emergency department because of a 4-hour history of a severe headache, anxiety, sweating, and palpitations. He has a 2-year history of hypertension well controlled with clonidine and hydrochlorothiazide, but he ran out of his prescriptions 3 days ago. His pulse is 120/min, and blood pressure is 170/110 mm Hg. Physical examination shows diaphoresis. Which of the following effects is most likely to follow an injection of propranolol in this patient? A) Decreased total peripheral resistance and decreased cardiac output B) Decreased total peripheral resistance and increased cardiac output C) Increased total peripheral resistance and decreased cardiac output D) Increased total peripheral resistance and increased cardiac output E) No change in total peripheral resistance or cardiac output

C) Increased total peripheral resistance and decreased cardiac output

A healthy 23 year old woman engages in strenuous aerobic exercise for 30 minutes. Which of the following best describes the total cross sectional area of the pulmonary microcirculation in this woman during exercise? A) Decreased, as a result of increased carbon dioxide levels in blood B) Decreased, in proportion to the increased resistance C) Increased, as a result of distention and recruitment of microvessels D) Increased, as a result of myogenic reflexes E) Unchanged

C) Increased, as a result of distention and recruitment of microvessels

A 5-year-old girl is brought to the physician because of increasingly severe pain of her left leg 1 day after she fell off a sled. She recently immigrated to the USA from Iceland with her family. She has a history of three bone fractures since birth. Her parents are vegan and do not give the patient any dairy products. Physical examination shows tenderness, swelling, and limited range of motion of the left lower extremity. An x-ray of the left lower extremity shows a fracture of the fibula, This patient most likely has a vitamin deficiency that directly affects which of the following processes? A) Collagen synthesis B) Folate metabolism C) Intestinal calcium absorption D) Osteoclastic activity E) Parathyroid hormone production

C) Intestinal calcium absorption

A 36-year-old man who is heterozygous for an LDL-receptor deficiency has a total serum cholesterol concentration of 330 mg/dL, After taking a drug that inhibits cholesterol synthesis, his total serum cholesterol concentration decreases to 200 mg/dL. Which of the following proteins would be upregulated as a result of drug therapy? A) Cholesterol acyltransferase B) Hepatic lipase C) LDL receptor D) Lipoprotein lipase E) Lysosomal cholesterol esterase

C) LDL receptor

23. Twenty subjects participate in a study of cardiopulmonary physiology. The pulmonary circulation of these subjects most likely has which of the following characteristics compared with their bronchial circulation? A) Greater number of branches supplying the visceral pleura B) Larger number of anastomoses with the intercostal arteries C) Larger percentage of cardiac output D) Larger volume of nutrient blood to the conducting airways E) Lower-volume, higher-pressure system

C) Larger percentage of cardiac output

A 58-year-old man comes to the physician because of a 4-year history of recurrent cough productive of increased sputum, Use of over-the-counter cough suppressants has not resolved his symptoms. He has smoked 2 packs of cigarettes daily for 35 years. He has no family history of lung disease, His temperature is 37°C (98.6'F), pulse is 72/min, and respirations are 18/min, Physical examination shows cyanosis. Diffuse wheezing is heard on auscultation. Which of the following pulmonary cell types is most likely to be abnormal in this patient? A) Alveolar endothelial cells B) Alveolar macrophages C) Pseudostratified columnar epithelial cells D) Type I pneumocytes E) Type II pneumocytes

C) Pseudostratified columnar epithelial cells

37. A 20-year-old man comes to the physician because of a 6-month history of diarrhea and a 16-kg (35-IP) weight loss. He is 186 cm (6 ft 1 in) tall and now weighs 75 kg (166 lb); BMI is 22 kg/m2. Physical examination shows pallor and pitting edema of the lower extremities. Laboratory studies show microcytic hypochromic anemia, hypoalbuminemia, and an increased fat concentration in stool. Serum anti-endomysium lgA and anti-tissue transglutaminase lgA antibodies are positive. A biopsy specimen of the small intestine is most likely to show which of the following in this patient? A) Eosinophil infiltration of lamina propria and mucosa B) Granulomas in bowel wall and serosa C) Loss of villi and increased number of intraepithelial lymphocytes D) Macrophages containing periodic acid-Schiff-positive bacilli in lamina propria E) Normal villi with epithelial cells vacuolated with fat

C) Loss of villi and increased number of intraepithelial lymphocytes

1. A newborn delivered at 28 weeks' gestation is in severe respiratory distress. The immature alveoli of this newborn's lungs have a diminished ability to serve as sites of effective gas exchange. An increase in which of the fallowing parameters best explains this finding? A) Alveolar radii B) Lung compliance C) Lung elastic recoil D) Pleural pressure E) Surfactant secretion

C) Lung elastic recoil

An otherwise healthy 23-year-old woman comes to the physician because she has not had a menstrual period for 2 months, Menarche occurred at the age of 14 years. and menses had occurred at regular 28-day intervals. Pelvic examination shows blue-tinged vaginal mucosa. The uterus is somewhat soft in the lower segment and is about the size of an orange. The adnexa are nontender with normal-sized ovaries. Which of the following is the most appropriate next step to establish the diagnosis? A) Measurement of serum follicle-stimulating hormone concentration B) Measurement of serum thyroid-stimulating hormone concentration C) Measurement of urine ß-human chorionic gonadotropin concentration D) Ultrasonography of the pelvis E) Endometrial biopsy

C) Measurement of urine ß-human chorionic gonadotropin concentration

A 65-year-old man with hypertension volunteers to participate in a clinical trial of a newly developed low diuretic, As pert of the study, his acid-base/volume status is monitored. After 3 days of treatment, which of the following sets of findings is most likely in this patient? Acid-base Volume Contraction A) Metabolic acidosis yes B) Metabolic acidosis no C) Metabolic alkalosis yes D) Metabolic alkalosis no

C) Metabolic alkalosis yes

A 40-year-old man has had orthostatic hypotension and loose stools for 1 year, He has a 26-year history of type 1 diabetes mellitus. Studies of the stool show no abnormalities. Which of the following pathophysiologic mechanisms is the most likely cause of the diarrhea? A) Exudation B) Generalized malabsorption C) Motility disorder D) Osmosis E) Secretion

C) Motility disorder

20. During an experiment of muscle contraction, intracellular calcium is decreased after a substance is administered to a muscle preparation obtained from an experimental animal. Which of the following best explains why contraction is inhibited in this case? A) Acetylcholine release is increased B) Depolarization along T tubules is enhanced C) Myosin binding sites on actin remain covered by troponin C D) Sodium influx is increased E) Tropomyosin is detached from actin

C) Myosin binding sites on actin remain covered by troponin C

A-19-year-old man who plays college basketball comes to the physician because of a 3-week history of shortness of breath with exertion, He says that he has been unable to complete his daily practice sessions. He also has had intermittent dizziness during this period. His pulse is 60/min, respirations are 18/min, and blood pressure is 100/60 mm Hg. Physical examination shows no abnormalities. Echocardiography shows a heterogeneous pedunculated mass in the left atrium. Which of the following is the most likely diagnosis? A) Hemangiopericytoma B) Lipoma C) Myxoma D) Rhabdomyosarcoma E) Teratoma

C) Myxoma

A 25-year-old woman comes to the physician because of palpitations and insomnia for 5 months: she also has had a 5-kg (11-lb) weight loss during this period. Her pulse is 125/min, Physical examination shows exophthalmos, thyromegaly, and hand tremors. Serum studies show a thyroid-stimulating hormone concentration of less than 0.1 µU/mL, thyroxine (T4) concentration of 18 µg/dL, and triiodothyronine (T3) concentration of 250 ng/dL. The signs and symptoms in this patient are most likely caused by hormones acting on which of the following receptors? A) Cytokine-linked kinase B) G protein-coupled receptor C) Nuclear/retinoid X binding to DNA D) Serine/threonine kinase E) Tyrosine kinase

C) Nuclear/retinoid X binding to DNA

A 44 year old man has the following findings on pulmonary function testing: Predicted Patient values Functional residual capacity (L) 3.2 4.0 Residual volume (L) 1.5 3.0 Total lung capacity (L) 5.9 6.8 Forced expiratory volume in 1 sec (FEV1) (L) 3.5 2.0 FEV1/forced vital capacity 75% 53% Forced expiratory flow (25-75%)(FEF 25-75%)(L) 4.4 1.2 This pattern of pulmonary function represents which of the following? A) Normal pulmonary function B) Obstructive and restrictive pulmonary disease C) Obstructive pulmonary disease only D) Restrictive pulmonary disease only

C) Obstructive pulmonary disease only

34. A 30-year-old man develops urinary incontinence 2 weeks after successful treatment of a fracture of the left pelvis that was sustained at work. He was pinned against a loading dock by a truck moving in reverse. Physical examination shows a distended bladder. Cystometrography shows absence of micturition reflexes. After the bladder fills to capacity. overflow of urine occurs through the urethra a few drops at a time. This patient most likely sustained additional injury to which of the following during his initial accident? A) External urinary sphincter B) Hypogastric nerve C) Pelvic nerves D) Pudendal nerves E) Skeletal motor fibers

C) Pelvic nerves

A 15-year-old Native American boy is brought to the emergency department 1 hour after he ingested a broth made from jimsonweed, He has hallucinations. His pulse is 128/min. Physical examination shows flushing and dry mouth, He is disoriented to person, place, and time, The pharmacologic antidote must be carefully titrated to avoid potential cardiac effects. Which of the following drugs is the most appropriate antidote? A) Carbachol B) Neostigmine C) Physostigmine D) Pilocarpine E) Trimethaphan

C) Physostigmine

During an experiment, a researcher blocks the production of microtubule-associated (MAP-1C) ATPase (dynein) in the neuronal cell body. This is most likely to cause a reduction of which of the following neuronal processes? A) Fast anterograde axonal transport B) Neurotransmitter reuptake C) Retrograde axonal transport D) Slow anterograde axonal transport E) Vesicle release

C) Retrograde axonal transport

A 56 year old man is brought to the hospital because of lethargy for the past month. He is not taking any medications. He does not have postural hypotension or edema. Laboratory studies show: Serum Na+ 125 mEq/L Creatinine 1 mg/dL Thyroxine (T4) 10 µg/dL Cortisol (AM) 20 µg/dL Plasma osmolality 275 mOsmol/kg Urine osmolality 450 mOsmol/kg Urine Na+ 30 mEq/L These findings are most likely associated with which of the following neoplasms? A) Adrenocortical carcinoma B) Renal cell carcinoma C) Small cell carcinoma of the lung D) Transitional cell carcinoma of the bladder E) Villous adenoma of the colon

C) Small cell carcinoma of the lung

A 60 year old woman comes to the physician because of a 2 day history of flank pain and pain with urination. She has hypertension, recurrent urinary tract infections, and hypothyroidism. Her temperature is 37C(98.6F), pulse is 96/min, and blood pressure is 152/92 mm Hg. Physical examination shows no other abnormalities. Urinalysis shows 3+ blood, 1+ leukocytes, and few bacteria. Ultrasonography of the abdomen shows a large calculus filling the entire right renal pelvis. The renal calculus in this patient is most likely composed of which of the following substances? A) Calcium oxalate B) Cystine C) Struvite D) Uric acid

C) Struvite

A 48-year-old man comes to the physician because he is concerned about his weight, He is 178 cm (5 ft 10 in) tall and weighs 91 kg (200 lb): BMI is 29 kg/m2. His waist circumference is 103 cm (41 in). His blood pressure is 140/90 mm Hg. Physical examination shows a protuberant abdomen. Laboratory studies show fasting serum glucose and insulin concentrations that are increased and remain increased 2 hours after the patient receives 75 g of oral glucose. Which of the following sets of additional laboratory findings in serum is most likely in this patient? Triglycerides HDL-cholesterol Free Fatty Acids A) UP UP UP B) UP UP DOWN C) UP DOWN UP D) DOWN UP DOWN E) DOWN DOWN UP F) DOWN DOWN DOWN

C) UP DOWN UP

2. An animal study is conducted to assess the effects of smoking on pulmonary defense and maintenance mechanisms. For 1 week, normal male rats are exposed to levels of cigarette smoke comparable to those encountered by humans who smoke cigarettes. Results of pulmonary testing are compared withbaseline levels obtained the week before the smoke exposure. Which of the following sets of changes is most likely to be observed? Mucus Production and Secretion; Activity of Airway Cilia; Alveolar Macrophage Function A) UP; UP; UP B) UP; UP; DOWN C) UP; DOWN; DOWN D) DOWN; UP; UP E) DOWN; DOWN; UP F) DOWN; DOWN; DOWN

C) UP; DOWN; DOWN

A 12-year-old boy is swimming in a mountain stream. He is immersed up to his neck in 60°F water for 20 minutes, Which of the following sets of physiologic changes is most likely to occur in this boy? Serum: Central Blood Volume; ADH (vasopressin); Atrial Natriuretic Peptide A) UP; UP; UP B) UP; UP; DOWN C) UP; DOWN; UP D) UP; DOWN; DOWN E) DOWN; UP; UP F) DOWN; UP; DOWN G) DOWN; DOWN; UP H) DOWN; DOWN; DOWN

C) UP; DOWN; UP

A previously healthy 25 year old man is brought to the emergency department because of severe pain on the left side of his chest after falling from a ladder. The pain is sharp and stabbing and becomes more intense when the patient coughs or takes a deep breath. An x-ray of the chest shows fractures of four consecutive ribs on the left. There is no pneumothorax. Which of the following patterns represents the most likely changes in respiratory status caused by his injury? Respiratory Rate Tidal Volume Airway Resistance (1 sec) A) ^ ^ ^ B) ^ <-> ^ C) ^ down <-> D) <-> <-> <-> E) down ^ down F) down down <-> G) down <-> down

C) ^ down <->

5. During a study of pulmonary artery pressure, subjects with hypertension experience a decrease in this pressure after receiving a phosphodiesterase inhibitor. This improvement in pulmonary artery pressure is most likely mediated by an increase in the intracellular concentration of which of the following? A) Adenosine B) Calcium C) cGMP D) Endothelin E) Nitric oxide

C) cGMP

To decrease his risk for cardiovascular disease, a 24-year-old man begins a weight-loss diet. He is 153 cm (5 ft 8 in) tall and weighs 95 kg (210 lb); BMI is 32 kg/m2. He intends to lose 16 kg (35 lb) by limiting his daily caloric intake to 2000 calories. In order to maintain the recommended protein intake (56 g daily), a balanced decrease in carbohydrates and fat is required (caloric ratio of fat to carbohydrate is 30:55), Which of the following best describes the number of calories that should be provided by fat in this patient's diet each day? A) 370 B) 430 C) 510 D) 630 E) 740

D) 630

9. A 59-year-old man undergoes a total thyroidectomy and central lymph node dissection for a 4-cm follicular carcinoma of the thyroid. Twelve hours after the procedure, he has paresthesias of his hands and feet. Vital signs are stable, but carpal spasm is noted on inflation of the blood pressure cuff. Which of the following sets of laboratory findings in serum is most likely in this patient? Parathyroid Hormone Calcium A) UP UP B) UP DOWN C) NORMAL NORMAL D) DOWN UP E) DOWN DOWN

E) DOWN DOWN

41. During an experiment, Drug X is administered to treat trematode and cestode infestations, Results show that Drug X increases the permeability of the cell membranes to calcium, causing paralysis, dislodgement, and death of the parasite, Drug X most closely resembles which of the following? A) Albendazole B) Bithionol C) Diethylcarbamazine D) Niclosamide E) Praziquantel

E) Praziquantel

Ten years after a total gastrectomy, a 60-year-old man has difficulty walking. Mentation and cranial nerve function are normal. He has diffuse spasticity in his arms and legs. markedly impaired proprioception in his feet, increased muscle stretch reflexes in his arms and knees, absence of muscle stretch reflexes in the ankles, and bilateral extensor plantar responses. The most likely cause is deficiency of which of the following vitamins? A) A B) B1 (thiamine) C) B6 (pyridoxine) D) B12 (cobalamin) E) Biotin F) C G) E H) K2 I) Retinol

D) B12 (cobalamin)

43. A 57-year-old man with alcoholism has a distended abdomen with shifting dullness and a fluid wave, He has caput medusae, palmar erythema, and spider angiomata. Which of the following is the most likely additional finding? A) Digital clubbing B) Enlarged inguinal lymph nodes C) Flame-shaped retinal hemorrhages D) Gynecomastia E) Jugular venous distention

D) Gynecomastia

A healthy 25-year-old man lives at 10,000 feet above sea level for 4 weeks. Two weeks after returning to sea level, ventilation has returned to normal, but oxygen delivery to muscles is greater than before acclimatization, An increase in which of the following best explains the oxygen delivery finding in this man? A) Arterial pH B) Arterial Po2 C) Cardiac output D) Hematocrit E) Pulmonary artery pressure

D) Hematocrit

10. A 55-year-old man is admitted to the hospital for treatment of sepsis. He appears anxious and confused. Treatment with vancomycin and ceftriaxone was initiated in the emergency department. His temperature is 39.8°C (103.6°F), pulse is 132/min, respirations are 28/min, and blood pressure is 85/48 mm Hg. Physical examination shows warm, flushed skin that is moist with diaphoresis_ There is no edema. It is most appropriate to administer which of the following solutions at this time? A) 5% Dextrose in water and 0.45% saline B) 5% Dextrose in water only C) 0.45% Saline only D) 0.9% Saline E) 3% Saline

D) 0.9% Saline

17. A 4-day-old boy is brought to the pediatrician's office early in the morning because he had vomited throughout the night after breast-feeding. Physical examination is normal except for lethargy and dry mucous membranes. Laboratory studies show: Serum: Na+ 139 mEq/L, Cl- 90 mEq/L, K+ 7.0 mEq/L, HCO3- 17 mEq/L, Glucose 42 mg/dL, Urea nitrogen (BUN) 25 mEq/dL, Creatinine 0.4 mg/dL, 17-Hydroxyprogesterone increased The most likely explanation for these findings is a deficiency in which of the following enzymes? A) 20,22-Desmolase B) 11ß-Hydroxylase C) 17α-Hydroxylase D) 21-Hydroxylase E) 17-Ketoreductase

D) 21-Hydroxylase

A previously healthy 35-year-old woman develops hypoxemia 35 minutes after ingesting a near-lethal dose of barbiturates. She has not aspirated. Which of the following sets of arterial blood gas values (in mm Hg) is most likely in this patient? Po2 Pco2 (A—a)O2 A) 40 50 35 B) 40 60 40 C) 50 25 10 D) 50 80 10 E) 60 35 25

D) 50 80 10

A 48 year old man comes to the physician for a follow-up examination. 3 years ago he underwent operative removal of his right kidney and adrenal gland for well-encapsulated renal cell carcinoma. His 2 previous annual examinations since then showed no abnormalities. He does not drink alcohol or smoke cigarettes. His pulse is 75/min and blood pressure is 130/88 mm Hg. Physical examination shows no abnormalities. Serum studies show mildly increased total bilirubin concentration and alkaline phosphatase, AST, and ALT activities. A CT scan of the abdomen shows widening of hepatic venules. Catheters guided by radiographic fluorography to measure pressures across the liver show: Right atria 5 mm Hg, Hepatic vein 25 mm Hg, and Hepatic wedge 30 mm Hg of pressures. Further evaluation is most likely to show which of the following? A) Acute cholecystitis B) Acute hepatitis C) Acute pancreatitis D) Budd-Chiari syndrome E) Cirrhosis

D) Budd-Chiari syndrome

A full-term female newborn develops respiratory distress shortly after delivery. Physical examination shows cyanosis, markedly decreased breath sounds bilaterally, and an occasional bowel sound over the left hemithorax. A chest x-ray taken while the patient is supine shows multiple cystic-appearing areas on the left and decreased aeration on the right Which of the following is the most likely diagnosis? A) Bacterial pneumonia B) Bullae-related pneumothorax C) Congenital cystic adenomatoid malformation D) Congenital diaphragmatic hernia E) Laryngeal atresia F) Lobar sequestration G) Tracheal stenosis

D) Congenital diaphragmatic hernia

44. A 55-year-old woman comes to the physician because of a 3-month history of shortness of breath. She takes a daily multivitamin. Her pulse is 98/min, respirations are 28/min and blood pressure is 133/88 mm Hg, Scattered crackles are heard over the lung bases. Laboratory studies show: Leukocyte count 8000/mm3, Serum: Ca2+ 12 mg/dL, Urea nitrogen 10 mg/dL, Creatinine 1 mg/dL, Phosphorus 4.2 mg/dL. An x-ray of the chest shows enlarged hilar lymph nodes and reticulonodular infiltrates in the upper lung fields. Analysis of a biopsy specimen of one of the enlarged lymph nodes shows noncaseating granulomas. Which of the following additional serum findings are most likely in this patient? Parathyroid Hormone 1,25- Dihydroxycholecalciferol A) UP, UP B) UP, NORMAL C) UP, DOWN D) DOWN, UP E) DOWN, NORMAL F) DOWN, DOWN

D) DOWN, UP

A 65-year-old woman comes to the physician for a follow-up examination after a DEXA scan showed a decrease in bone density. Physical examination shows no abnormalities. Which of the following sets of findings in bone is most likely in this patient? Osteoblast Activity; Osteoclast Activity; Receptor Activator of Nuclear Factor KB Ligand (RANKL) A) UP; UP; UP B) UP; DOWN; UP C) UP; DOWN; DOWN D) DOWN; UP; UP E) DOWN; UP; DOWN F) DOWN; DOWN; DOWN

D) DOWN; UP; UP

18. Which of the following best explains why deoxygenated blood can carry more carbon dioxide for a given Pco2 than oxygenated blood? A) Deoxyhemoglobin does not bind to 2,3-bisphosphoglycerate as efficiently as oxyhemoglobin B) Deoxyhemoglobin has a lower capacity to form carbamino compounds than oxyhemoglobin C) Deoxyhemoglobin has a lower pKa than oxyhemoglobin D) Deoxyhemoglobin is a better buffer of hydrogen ions than oxyhemoglobin E) Oxygen and carbon dioxide compete for the same binding site in hemoglobin F) Oxyhemoglobin binds nitric oxide with a higher affinity than deoxyhemoglobin

D) Deoxyhemoglobin is a better buffer of hydrogen ions than oxyhemoglobin

A 28-year-old woman is brought to the hospital by her boyfriend because of severe lethargy for 2 hours, Her temperature is 37.1°C (98,7°F), pulse is 110/min, respirations are 24/min. and blood pressure is 126/78 mm Hg. Physical examination shows obtundation, Her serum glucose concentration is 25 mg/dL, She revives immediately upon intravenous administration of 50% dextrose in water. She does not have type 1 or type 2 diabetes mellitus, and she takes no medications. Multiple nonfasting glucose concentration measurements are within the reference range, and results of a 75-g, 2-hour glucose tolerance test show no abnormalities. Corresponding serum insulin concentrations are within the reference range. Six hours later, she is unresponsive, Her serum glucose concentration is 32 mg/dL. She is again resuscitated with 50% dextrose in water. The physician suspects surreptitious use of insulin. Which of the following is the most appropriate next step? A) Confront the patient about the physician's suspicion B) Search the patients personal belongings for insulin and syringes C) Obtain a psychiatric consultation D) Determine the C-peptide concentration in the latest blood specimen drawn E) Measure hemoglobin A1c

D) Determine the C-peptide concentration in the latest blood specimen drawn

A 50-year-old woman comes to the physician because of progressive shortness of breath during the past 2 years. Her respirations are 20/min. Physical examination shows cyanosis and ankle edema, Her pulmonary artery pressure, pulmonary vascular resistance, and right atrial pressure are increased; her pulmonary capillary wedge pressure is 9.3 mm Hg (N=8-16), Her ventilation improves when inhaled nitric oxide is administered. Treatment with oral bosentan is begun for long-term therapy. Which of the following mediators of pulmonary resistance in this patient will most likely be antagonized by this drug? A) Adenosine B) Calcitonin gene-related peptide C) C-reactive protein D) Endothelin E) Prostacyclin (PGI2)

D) Endothelin

A previously healthy 48-year-old man comes to the physician because of a 2-month history of increasing abdominal girth and inability to achieve an erection. He has smoked 1 pack of cigarettes daily for 20 years; he also has drunk 1 pint of liquor daily during this period. His temperature is 37°C (98.6°F). pulse is 98/min. respirations are 14/min, and blood pressure is 120/76 mm Hg, Physical examination shows scleral icterus and spider angiomata. The lungs are clear to auscultation. Cardiac examination shows no abnormalities, There is gynecomastia. Abdominal examination shows ascites and a prominent umbilical venous pattern. His testes are small. Which of the following is the most likely cause of this patients gynecomastia? A) Excessive estrogen production by the adrenal glands B) Excessive testosterone degradation by the liver C) Failure of the liver to conjugate testosterone to its carrier molecule D) Failure of the liver to degrade estrogen E) Inadequate gonadotropin secretion by the pituitary gland F) Inadequate testosterone production by the adrenal glands

D) Failure of the liver to degrade estrogen

A 33-year-old man comes to the physician because of a 3-month history of muscle weakness and cramping. He says that the symptoms appear shortly after he begins exercising. Physical examination shows no abnormalities. Serum creatine kinase concentration is increased, After the forearm muscles are exercised, venous blood obtained from the antecubital vein shows that lactate concentrations do not increase compared with preexercise values, This patient most likely has a deficiency in the activity of which of the following enzymes in muscle? A) Carnitine palmitoyltransferase-1 B) Fumarase C) Glucose 6-phosphatase D) Glycogen phosphorylase E) Succinate dohydrogenase

D) Glycogen phosphorylase

A 63-year-old man comes to the physician because of a 3-month history of difficulty sleeping. He indicates that he sleeps better if he sits upright in bed. His pulse is 90/min, and blood pressure is 110/60 mm Hg. Physical examination shows increased jugular venous pressure and mild ankle edema. Which of the following is the most likely cause of the edema in this patient? A) Decreased capillary hydrostatic pressure B) Decreased capillary permeability C) Decreased plasma colloid osmotic pressure D) Increased capillary hydrostatic pressure E) Increased capillary permeability F) Increased plasma colloid osmotic pressure

D) Increased capillary hydrostatic pressure

A 48-year-old woman comes to the emergency department because of palpitations. Pulse is 180/min, and blood pressure is 104/68 mm Hg. ECG shows an atrioventricular (AV) nodal re-entrant tachycardia. Carotid sinus massage is performed and sinus rhythm is restored. Which of the following is the most likely mechanism of the termination of the tachycardia in this patient? A) Activation of carotid chemoreceptors B) Blockade ß-adrenergic receptors in the AV node C) Blockade of L-type calcium channels in the AV node D) Increased cardiac parasympathetic activity E) Increased cardiac sympathetic activity

D) Increased cardiac parasympathetic activity

8. A 48-year-old woman comes to the physician because of a 6-month history of irregular menstrual periods and hot flashes. Her last menstrual period was 35 days ago, and she had scant blood flow, Menses had previously occurred at regular 28-day intervals. Physical examination shows mild thinning of the vaginal tissue, Laboratory studies are most likely to show which of the following serum concentrations in this patient? A) Decreased adrenocorticotropic hormone B) Decreased luteinizing hormone C) Decreased thyroid-stimulating hormone D) Increased follicle-stimulating hormone E) Increased prolactin

D) Increased follicle-stimulating hormone

A previously healthy 40-year-old man is brought to the emergency department 1 hour after the sudden onset of severe pain in his left leg while playing tennis. He is found to have ruptured the left Achilles tendon and undergoes operative repair and long leg cast immobilization, Six months later, the left calf shows a 2-cm decrease in circumference compared with the right calf. Which of the following is the most likely cause of this decrease? A) Decreased glycogen synthesis B) Decreased myosin light chain phosphatase activity C) Increased phosphatidyl degradation D) Increased protein degradation E) Mitochondria damage F) Necrosis of muscle fibers

D) Increased protein degradation

38. A 40-year-old man with interstitial pulmonary fibrosis has a greater maximal expiratory flow rate than predicted. Which of the following best explains this finding? A) Hypercapnic bronchodilatation B) Hyperinflation of the lungs C) Hypoxic bronchodilatation D) Increased radial traction on airways E) Increased surfactant secretion

D) Increased radial traction on airways

A 20-year-old man is brought to the physician because of a 4-hour history of abdominal pain, nausea, and vomiting. He says that he had been drinking ethanol heavily all weekend, and he took three doses of acetaminophen within 2 hours after the onset of a severe headache Monday morning. This patient is at increased risk for liver injury because of which of the following actions of ethanol? A) Activation of IgE-mediated mast cell degranulation B) Decreased acetaminophen clearance via glucuronidation C) Increased bioavailability of acetaminophen D) Induction of cytochrome P450 enzymes that activate acetaminophen to a hepatotoxic metabolite E) Metabolic acidosis due to an increased NADH:NAD+ ratio

D) Induction of cytochrome P450 enzymes that activate acetaminophen to a hepatotoxic metabolite

30. A 68-year-old man comes to the physician because of a 6-month history of erectile dysfunction. Physical examination and laboratory evaluation show no abnormalities. If pharmacotherapy is indicated, the most appropriate is a drug with which of the following mechanisms of action? A) Blockade of α2-adrenergic receptors B) Increased activity of 5α-reductase C) Inhibition of nitric oxide release D) Inhibition of phosphodiesterase E) Stimulation of ß1-adrenergic receptors

D) Inhibition of phosphodiesterase

A 76-year-old man with congestive heart failure comes to the physician because of a 1-month history of shortness of breath after walking up a flight of stairs. He has had frequent nausea, and he has had to use two pillows to sleep during this period. Diffuse, moist crackles are heard over both lungs. There is pitting edema above the ankles. Which of the following best describes the function of the product secreted by the atrial myocyles in this patient? A) Decreased glomerular filtration rate B) Increased sodium reabsorption C) Induction of vasoconstriction D) Inhibition of renin release E) Stimulation of aldosterone release

D) Inhibition of renin release

A 33-year-old woman comes to the physician for a follow-up examination. Three weeks ago, she underwent oophorectomy because of epithelial ovarian cancer. At today's visit, the physician recommends adjuvant chemotherapy including paclitaxel. Which of the following best describes the mechanism of action of paclitaxel in this patient? A) Alkylates DNA B) Destroys asparagine C) Inhibits dihydrofolate reductase D) Inhibits microtubule disassembly E) Prevents microtubule polymerization

D) Inhibits microtubule disassembly

A 79-year-old man is brought to the emergency department by his wife 30 minutes after he lost consciousness for 30 seconds, On arrival, he is alert, but he says that he is dizzy. There is no urinary or fecal incontinence. His pulse is 40/min and regular, and blood pressure is 92/56 mm Hg. Physical examination shows no evidence of tongue biting. The lungs are clear to auscultation, Cardiac examination shows a variable intensity S1, He is oriented to person, place, and time. An ECG shows a third-degree atrioventricular block, Which of the following is the most appropriate next step in management? A) Transesophageal echocardiography B) Transthoracic echocardiography C) Insertion of pulmonary artery catheter D) Insertion of transvenous pacemaker E) Cardiac catheterization with angioplasty F) Cardiac catheterization with stent placement

D) Insertion of transvenous pacemaker

An 18-year-old woman comes to the physician because of progressive fever, general malaise, and blood in her urine since she began oral antibiotic therapy for a urinary tract infection 5 days ago. She also has a 3-day history of a rash, Her temperature is 38°C (100.4°F), pulse is 75/min, respirations are 12/min, and blood pressure is 125/80 mm Hg. Physical examination shows a petechial rash over the chest, back, and upper and lower extremities. Urinalysis shows: Blood 3+, Protein 1+, Leukocytes 150/hpf, Eosinophils 30%. Which of the following is the most likely diagnosis? A) Acute tubular necrosis B) Glomerulonephritis C) IgA nephropathy D) Interstitial nephritis E) Papillary necrosis

D) Interstitial nephritis

A 50-year-old man is admitted to the hospital after being diagnosed with a pulmonary embolus. Treatment is started with intravenous heparin. Twenty-four hours later, warfarin is added. On day 2, his partial thromboplastin time is 52 seconds (control 26 sec), and prothrombin time is 12 seconds (control 12.1 sec; INR=1), Which of the following is the best explanation for the normal prothrombin time and INR measurements in this patient? A) Heparin-warfarin interaction B) Hereditary resistance to heparin C) Hereditary resistance to warfarin D) Lang half-life of factor II (prothrombin) E) Too low a dose of heparin F) The low a dose of warfarin G) Undetected liver disease

D) Lang half-life of factor II (prothrombin)

6. A 4-year-old girl has had puffy eyes for the past 3 days. She has had an upper respiratory tract infection for the past week. A urine dipstick test shows 3+ protein. Urinalysis shows no red or white blood cells or casts_ Which of the following mechanisms is the most likely cause of this patient's condition? A) Attack by antiglomerular basement membrane antibodies B) Damage to the loop of Henle C) Damage to the proximal renal tubule D) Loss of negative glomerular charge E) Presence of a posterior urethral valve

D) Loss of negative glomerular charge

A previously healthy 17-year-old girl is brought to the emergency department because of a 1-day history of shortness of breath, weakness, and muscle tenderness, She completed a triathlon the previous day. She appears restless. She is 163 cm (5 ft 4 in) tall and weighs 50 kg (110 lb); BMI is 19g kg/m2. Her temperature is 38°C (100.4F), respirations are 20/min, and blood pressure is 160/90 mm Hg. Bilateral crackles are heard in the lower lung lobes on auscultation of the chest. Physical examination shows muscle tenderness, Her serum creatinine concentration is 4 mg/dL, Urinalysis shows 3+ protein and 4+ hemoglobin. The most likely cause of this patient's condition is an increased release of which of the following substances? A) Aldolase B) Creatine kinase C) Hemoglobin D) Myoglobin E) Troponin I

D) Myoglobin

A 35-year-old woman, gravida 1, para 1, comes to the physician because of a 2-month history of generalized weakness and fatigability that are significantly exacerbated by exercise, The symptoms began during her pregnancy. Physical examination shows diplopia. There is reduction in arm muscle strength with repetitive movement, but preservation of deep tendon reflexes, A therapeutic trial of oral pyridostigmine provides symptomatic relief. Serum studies will most likely show antibodies to which of the following proteins? A) Acetylcholinesterase B) Muscarinic acetylcholine receptor C) Myeloperoxidase D) Nicotinic acetylcholine receptor

D) Nicotinic acetylcholine receptor

A healthy 22-year-old man is a subject in a study investigating the regulation of heart rate during exercise. He is sitting on a stationary bicycle and is instructed to begin pedaling in 5 minutes. One minute before he begins to pedal, his heart rate increases. An increase in which of the following transmitters acting at the indicated receptor best explains the increase in heart rate in this man? Transmitter Receptor A) Acetylcholine muscarinic B) Acetylcholine nicotinic C) Norepinephrine α D) Norepinephrine ß1 E) Norepinephrine ß2

D) Norepinephrine ß1

An experimental model of treatment for sickle cell disease involves reactivating the genes that code for the ß chains of fetal hemoglobin, This treatment is most likely to increase the affinity of hemoglobin for which of the following? A) 2,3-Bisphosphoglycerate B) Carbon dioxide C) Chloride ions D) Oxygen E) Protons

D) Oxygen

A 70-year-old man has a 4-month history of weight loss, abdominal pain, and diarrhea, Stool analysis shows increased excretion of neutral fat and muscle fiber. A d-xylose test for carbohydrate absorption shows no abnormalities. Examination of tissue obtained on intestinal biopsy shows no abnormalities. This patient is most likely to respond favorably to administration of which of the following agents? A) Antibiotics B) Azathioprine C) Intrinsic factor D) Pancreatic enzymes E) Prednisone

D) Pancreatic enzymes

20. A 60-year-old man comes to the physician because of a 6-month history of fatigue. Four years ago, he had a subtotal gastrectomy after he sustained a gunshot wound to the abdomen. He drinks six to eight beers daily. Physical examination shows paresthesias of both hands. Laboratory studies show: Hemoglobin 8 g/dL, Hematocrit 24%, Mean corpuscular volume 115 µm3, Leukocyte count 5000/mm3, Platelet count 165.000/mm3, RBC folic acid 500 ng/mL (N=125-600), Serum vitamin B12 (cobalamin) 10 pg/mL (N=160-195) The most likely cause of his anemia is the absence of which of the following? A) Cardiac glands B) Chief cells C) G (gastrin) cells D) Parietal cells E) Transcobalamin II

D) Parietal cells

A 35-year-old woman comes to the physician because of intermittent sharp chest pain that develops suddenly, is sometimes exacerbated by deep breathing, and can be decreased by leaning forward, She has a 2-month history of pain and swelling in her hands and knees accompanied by morning stiffness that usually lasts 1 hour. Vital signs are normal. Pulsus paradoxus is less than 10 mm Hg. Auscultation of the chest shows a harsh scratchy sound that is present in both systole and diastole. There is warmth, swelling, and tenderness of the metacarpophalangeal joints and Knees. Which of the following is most likely responsible for the chest pain in this patient? A) Costochondritis B) Ischemic heart disease C) Mitral valve prolapse D) Pericarditis E) Pulmonary fibrosis

D) Pericarditis

Following an operation, a 65-year-old patient has a lung region that is underventilated but well perfused. This condition will lead to an increase in which of the following? A) Alveolar dead space B) Anatomic dead space C) Physiologic dead space D) Physiologic shunt E) Po2

D) Physiologic shunt

42. A 28-year-old woman has hirsutism and irregular menstrual periods. Which of the following is the most likely underlying abnormality? A) Benign cystic teratoma of ovary B) Brenner tumor C) Dysgerminoma D) Polycystic ovarian disease E) Uterine endometriosis

D) Polycystic ovarian disease

A 20-year-old man comes to the physician because of a 3-month history of progressive thirst and urinary frequency; the thirst became excessive during the past 3 days. He says that he has been drinking approximately 10 quarts of ice water, in addition to approximately five large glasses of iced tea and lemonade, daily for 1 to 2 weeks. Physical examination shows no abnormalities, Urinalysis shows a specific gravity of less than 1.006. This patient most likely has dysfunction of which of the following endocrine structures? A) Adrenal cortex B) Adrenal medulla C) Anterior pituitary gland D) Posterior pituitary gland E) Thyroid follicular cells F) Thyroid parafollicular cells

D) Posterior pituitary gland

13. A previously healthy 24-year-old man is admitted to the hospital after having a generalized tonic-clonic seizure. He is somnolent. Serum studies show a sodium concentration of 118 mEcilL and a potassium concentration of 4.0 mEq/L. Urinalysis shows a specific gravity of 1.005. The sodium concentration slowly returns to the reference range over the next 36 hours after treatment. The patient remains somewhat somnolent and stays in bed. On the third hospital day, serum osmolality is 280 mOsmol/kg. and two random measurements of urine osmolality are 260 and 272 mOsmol/kg. respectively. The patient is more alert and active. He is observed drinking a large amount of water (more than 6 liters in 1 day), and his serum sodium concentration begins to decrease. When water restrictions are imposed, he drinks from water fountains on other floors. He says he does not have excessive thirst. Which of the following is the most likely cause of this patient's condition? A) Adrenal insufficiency B) Central diabetes insipidus C) Nephrogenic diabetes insipidus D) Psychogenic polydipsia E) Syndrome of inappropriate secretion of ADH (vasopressin)

D) Psychogenic polydipsia

24. A 62-year-old man is brought to the emergency department because of a 3-hour history of progressive difficulty breathing and mild left shoulder pain. His symptoms began after he ran up several flights of stairs. He has poorly controlled hypertension and mild angina pectoris. He has smoked one-half pack of cigarettes daily for 40 years. He appears uncomfortable and has labored breathing. Diffuse crackles, rhonchi, and scattered wheezing are heard on auscultation of the posterior lung fields. His arterial Po2 is 58 mm Hg. Which of the following is the most likely diagnosis? A) Cardiac tamponade B) Pneumonitis C) Pneumothorax D) Pulmonary edema E) Pulmonary embolism

D) Pulmonary edema

50. A male newborn delivered at 34 weeks' gestation by an uncomplicated spontaneous vaginal delivery develops respiratory distress 30 minutes later. His respirations are 40/min. Physical examination shows cyanosis and lower rib retractions with respiration, Chest x-rays show fine densities with a reticulogranular appearance bilaterally. Which of the following is the most likely cause of these findings? A) Erythroblastosis fetalis B) Extralobar sequestration C) Intracranial hemorrhage D) Pulmonary surfactant deficiency E) Situs inversus

D) Pulmonary surfactant deficiency

20. A 25-year-old man has significant blood loss after he sustains a fracture of his right tibia during a soccer match Prior to emergency treatment, which of the following is most likely to compensate for his decreased blood volume? A) Decreased secretion of renin B) Filtration of fluid from systemic capillaries C) Increased secretion of atrial natriuretic peptide D) Reabsorption of fluid into systemic capillaries E) Relaxation of systemic arterioles F) Relaxation of venous smooth muscle

D) Reabsorption of fluid into systemic capillaries

Two days after admission to the hospital because of a sickle cell disease crisis, a 24-year-old man suddenly develops blood in his urine and lank pain. Laboratory studies at the time of admission showed that his hematocrit was decreased to 11% from his normal baseline of 25%. Urinalysis shows gross blood, rare WBCs, and no WBC casts. Which of the following is the most likely cause of the hematuria in this patient? A) Glomerulonephritis B) Nephrolithiasis C) Prostatitis D) Renal papillary necrosis E) Transitional cell carcinoma

D) Renal papillary necrosis

A 48 year old man is admitted to the hospital for treatment of idiopathic pulmonary hypertension. The patient has experienced progressive dyspnea over the past 6 months. Cardiac catheterization shows a decrease in cardiac function. The most likely cause of the decreased cardiac function is an increase in which of the following? A) Left ventricular preload B) Right ventricular preload C) Left ventricular afterload D) Right ventricular afterload

D) Right ventricular afterload

48. A 75-year-old man comes to the emergency department because of a 2-day history of ringing of his ears, nausea, and fatigue. His temperature is 37°C (98.6'F), pulse is 100/min, respirations are 24/min, and blood pressure is 140/85 mm Hg. Physical examination shows mild epigastric tenderness. Arterial blood gas analysis on room air shows: pH 7.42, Pco2 30 mm Hg, Po2 95 mm Hg, 19 mEq/L. Which of the following is the most likely cause of this patient's condition? A) Intracardiac shunt B) Obstructive sleep apnea C) Pulmonary embolism D) Salicylate poisoning E) Severe kyphosis

D) Salicylate poisoning

47. A 23-year-old man has a 3-month history of mildly depressed mood, decreased energy, and dry skin. His thyroid gland is not palpably enlarged. Serum thyroxine (T4) and thyroid-stimulating hormone concentrations are decreased. Which of the following is the most likely diagnosis? A) Primary hyperthyroidism B) Secondary hyperthyroidism C) Primary hypothyroidism D) Secondary hypothyroidism

D) Secondary hypothyroidism

1. A young adult couple has been unable to conceive for the past 2 years. The woman has regular menstrual cycles and takes no contraceptives. A semen analysis shows: Quality: normal range,Quantity: normal range, Color: white, Fructose: 5% of normal content The most likely cause of this couple's infertility is deficient activity of which of the following in the man? A) Adenohypophysis B) Bulbourethral glands C) Prostate D) Seminal vesicles E) Testes

D) Seminal vesicles

48. A 25-year-old woman has recently developed nervousness and emotional lability. Her pulse is 110/min and blood pressure is 135/70 mm Hg. She has a wide-eyed, staring gaze; her skin is warm and moist; and the fingers of both hands show a fine tremor when extended. Her thyroid gland is diffusely enlarged. Which of the following is the most likely cause of the thyroid changes? A) Antithyroid cytotoxic T lymphocytes B) Thyroid antithyroglobulin autoantibodies C) Thyroid peroxidase (antimicrosomal) autoantibodies D) Thyrotropin receptor autoantibodies E) Thyroxine (T4) autoantibodies

D) Thyrotropin receptor autoantibodies

24. A 13-year-old girl is brought to the physician by her mother because of a 5-month history of behavioral problems. The mother states that her daughter alternates from being sad and socially withdrawn to being extremely angry and aggressive. After further evaluation, a diagnosis of bipolar disorder is made. Treatment with valproic acid, which inhibits histone deacetylase, is started. This drug is most likely to affect which of the following processes in this patient? A) rnRNA splicing B) Polyadenylation C) Post-translational processing D) Transcription E) Translation

D) Transcription

An investigator studying spermatogensis introduces a chemical that disrupts tight junctions between adjacent cells into the testes of experimental animals. The animals later develop an autoimmune response to products of germ cell division that have entered the circulation. Tight junctions have been disrupted between which of the following cells in these animals? A) Capillary endothelium B) Early spermatids C) Fibroblasts D) Leydig E) Sertoli

E) Sertoli

A 48-year-old man is brought to the emergency department because of difficulty opening his mouth and stiffness of his neck, shoulders, and back for 4 hours. He sustained a puncture wound to his arm 1 week ago. He has received no medical care for 30 years. Physical examination shows trismus and opisthotonos. Which of the following best describes the process in this patient? A) Blockade of inhibitory neurotransmitter release B) Cell death in central nervous system neurons C) Cell death in muscles D) Cell death in peripheral nervous system neurons E) Demyelinization

E) Demyelinization

A healthy 5-year-old boy is brought to the physician by his parents for a well-child examination, He has a history of motion sickness, Physical examination shows no abnormalities. The parents are planning a vacation to Australia, and they ask the physician if they should give their son diphenhydramine during the trip for his motion sickness. Which of the following is the most likely mechanism of action of this medication for motion sickness? A) Agonist at α1-adrenoreceptors B) Agonist at ß1-adrenoreceptors C) Agonist at N-methyl-D-aspartate receptors D) Antagonist at histamine-2 (H2) receptors E) Antagonist at muscarinic-3 (M3) receptors F) Antagonist at serotonin receptors

E) Antagonist at muscarinic-3 (M3) receptors

A 6-year-old boy is brought to the physician by his mother because of a 3-month history of pubic and axillary hair growth, On previous visits, he has been at the 50th percentile for height and weight. Pubic and axillary hair development is Tanner stage 2. Physical examination shows mild enlargement of the testes. Left untreated, which of the following combinations would best describe the likely course of growth in this patient? Height Percentile 1 Year From Now As an Adult A) 25th 25th B) 25th 75th C) 50th 25th D) 50th 75th E) 75th 25th F) 75th 75th

E) 75th 25th

36. A patient in the early stages of hemorrhagic shock is most likely to have which of the following? A) Decreased ventilatory rate due to inhibition of the respiratory center B) Flushing due to cutaneous vasodilation that occurs secondary to sweating C) Increased renal blood flow due to sympathetic activation D) Sweating due to parasympathetic stimulation of sweat glands E) A weak pulse due to decreased stroke volume

E) A weak pulse due to decreased stroke volume

5. A 65-year-old woman with ovarian cancer is treated with cyclophosphamide and other chemotherapeutic agents. Cyclophosphamide affects which of the following targets in this patient? A) Bcr-Abl tyrosine kinase B) Caspase 3 C) DNA histone deacetylase D) DNA polymerase α E) DNA replication F) DNA topoisomerase II G) Epidermal growth factor receptor tyrosine kinase

E) DNA replication

38. A 32-year-old man who is an international health worker is given primaquine as prophylaxis against malaria. He becomes jaundiced. Laboratory findings are consistent with hemolytic anemia. The mechanism most likely to be responsible for this blood disorder is hemolysis due to which of the following? A) Antibody directed against the drug bound to a protein of the erythrocyte membrane B) Antibody directed against the drug in the plasma with complement binding to the erythrocyte membrane C) Autoantibody induced by the drug D) Direct injury to the erythrocyte membrane E) Enzyme deficiency in the erythrocytes

E) Enzyme deficiency in the erythrocytes

An otherwise asymptomatic 52-year-old woman comes to the physician because of hot flashes. Her menses have been irregular for the past 6 months. Which of the following is the most likely physiologic cause of her symptoms? A) Decreased hypothalamic secretion of gonadotropin-releasing hormone B) Excessive conversion of androstenedione to estrone C) Excessive production of adrenal androgens D) Failure of the anterior pituitary to secrete gonadotropins E) Failure of the ovaries to secrete 17ß-estradiol

E) Failure of the ovaries to secrete 17ß-estradiol

A 54-year-old woman is admitted to the hospital with an acute myocardial infarction, At the time of admission, she has no physical signs of heart failure, and no heart murmur is detected. Two days later, she becomes acutely short of breath and diaphoretic. Her pulse is 100/min, respirations are 24/min, and blood pressure is 160/98 mm Hg. Crackles are heard bilaterally throughout both lung fields on auscultation of the chest, A murmur is heard on cardiac examination. Which of the following is the most likely murmur in this patient? A) Grade 2/6, diastolic decrescendo murmur heard best over the second and third left intercostal spaces B) Grade 2/6, rumbling diastolic murmur heard best 2 cm left of the sternal border at the fourth left intercostal space C) Grade 3/6, crescendo-decrescendo systolic murmur heard best at the second left intercostal space D) Grade 4/6, continuous systolic and diastolic murmur heard best along the left sternal border E) Grade 4/6, holosystolic murmur heard best over the lower left sternal border and the cardiac apex

E) Grade 4/6, holosystolic murmur heard best over the lower left sternal border and the cardiac apex

A 13 year old girl grew 7.5 cm (3 in) over the summer. Which of the following most likely accounts for increased intestinal absorption of calcium during this period? A) Calcitonin-mediated hypercalcemia B) Calcium-binding proteins in goblet cells C) Cortisol-induced transcriptions of calcium transporters D) Cyclic AMP generated in the enterocytes in response to parathyroid hormone E) Hormones derived from 7-dehydrocholesterol

E) Hormones derived from 7-dehydrocholesterol

A 55-year-old man comes to the physician for a routine health maintenance examination. His father and his older brother both died of a myocardial infarction at the ages of 50 and 55 years, respectively. The patient is 175 cm (5 ft 9 in) tall and weighs 89 kg (196 lb); BMI is 29 kg/m2. Physical examination shows no other abnormalities. serum studies show a total cholesterol concentration of 300 mg/dL, HDL-cholesterol concentration of 40 mg/dL, and HDL-cholesterol concentration of 230 mg/dL. Treatment with lovastatin is begun. This treatment is most likely to result in which of the following adaptive responses at the cellular level in this patient? A) Decreased hepatic expression of LDL-cholesterol receptors B) Decreased transcription of HMG-CoA reductase C) Increased mevalonic acid degradation D) Increased mevalonic acid synthesis E) Increased transcription of HMG-CoA reductase

E) Increased transcription of HMG-CoA reductase

A previously healthy 42-year-old Asian woman is brought to the emergency department because of a 24-hour history of nausea, vomiting, and progressive lethargy. She has smoked 1 pack of cigarettes daily for 25 years and drinks four glasses of wine daily. She uses high-dose acetaminophen daily for headaches. She does not use illicit drugs. She is 155 cm (5 ft 1 in) tall and weighs 50 kg (110 lb); BMI is 21 kg/m2. She is responsive to painful stimuli. Initial laboratory studies show increased hepatic aminotransferase. Which of the following effects of alcohol most likely contributed to this patient's condition? A) Decreased generation of N-acetyl-p-benzoquinoneimine B) Increased glucuronidation C) increased hepatic glutathione stores D) increased sulfation E) Induction of cytochrome P450

E) Induction of cytochrome P450

32. A 67-year-old man has urinary urgency after placement of a urinary bladder catheter during transurethral resection of the prostate for carcinoma. The most appropriate therapy is a drug with which of the following actions? A) Activation of adenosine receptors B) Activation of muscarinic receptors C) Activation of nicotinic receptors D) Inhibition of adenosine receptors E) Inhibition of muscarinic receptors F) Inhibition of nicotinic receptors

E) Inhibition of muscarinic receptors

An investigator is studying the regulation of pulmonary lymphatic flow using an animal model. Catheters are implanted in the femoral vein, pulmonary artery, and main lymphatic vessel draining the lungs in anesthetized, intubated, mechanically ventilated animals, Drugs may be injected directly into the pulmonary artery catheter and inspired gas mixtures altered at the ventilator; timed collection of lymph from the catheters is used to calculate lymphatic flow. Which of the following interventions will most likely increase the flow of pulmonary lymph in these animals? A) Administration of endothelin-1 into the pulmonary artery B) Administration of phenylephrine into the pulmonary artery C) Decreasing the inspired oxygen concentration from 21% to 10% D) Increasing the inspired carbon dioxide concentration from 0.3% to 3% E) Intravenous infusion of 0.9% saline for 5 minutes F) Intravenous infusion of 20% albumin solution (20 g/100 mL saline) for 5 minutes

E) Intravenous infusion of 0.9% saline for 5 minutes

A 52-year-old man is brought to the emergency department 30 minutes after the onset of chest pain and shortness of breath, He had played tennis all day, and he does not remember how much fluid he had consumed. His temperature is 36.1°C (98°F), pulse is 122/min, respirations are 28/min, and blood pressure is 90/50 Mm Hg. Physical examination shows dry skin and decreased capillary refill. An ECG and evaluation of cardiac enzymes show no abnormalities, Which of the following findings in the nephron best describes the tubular osmolality, compared with serum, in this patient? Proximal Tubule Macula Derma Medullary Collecting Duct A) Hypertonic hypertonic hypertonic B) Hypertonic hypertonic hypotonic C) Hypertonic hypotonic hypotonic D) Isotonic isotonic isotonic E) Isotonic hypotonic hypertonic F) Isotonic hypotonic hypotonic G) Hypotonic hypertonic hypertonic H) Hypotonic hypotonic hypertonic I) Hypotonic hypotonic hypotonic

E) Isotonic hypotonic hypertonic

A 64-year-old man with a 25-year history of alcoholism is brought to the emergency department by his wife because of a 1-day history of confusion. He is disoriented and disheveled, His pulse is 110/min, respirations are 20/min, and blood pressure is 100/64 mm Hg, Physical examination shows signs of dehydration and jaundice, and spider angiomata over the face and chest. There is a flapping, up-and-down motion of the hands when the upper extremities are outstretched horizontally. Abdominal examination shows distention with bulging flanks and shifting dullness, In addition to other appropriate pharmacotherapy, administration of oral neomycin is begun. Which of the following primary mechanisms of action is most likely to occur in this patient as a result of this drug treatment? A) Binding of ammonia and other hepatically cleared toxins in the gut B) Blockade of new protein synthesis by the liver C) Blockade of two successive steps in the metabolism of folic acid D) Increased endotoxin production where bacterial overgrowth occurs in the gut E) Killing of bacteria in the gut that generate ammonia

E) Killing of bacteria in the gut that generate ammonia

A 35-year-old man with Chagas disease comes to the emergency department because of a 2-hour history of moderate chest pain. He has had a 2-kg (4.4-1b) weight loss during the past 2 months. His pulse is 100/min, respirations are 25/min, and blood pressure is 135/90 mm Hg. An ECG shows no abnormalities. A barium swallow shows a dilated esophagus with beak-like narrowing at the level of the lower esophageal sphincter (LES). A biopsy specimen of the LES is most likely to show which of the following? A) Absence of anticholinesterase-like immunoreactivity B) Degeneration of motor end-plates C) Degeneration of smooth muscle D) Increased vasoactive intestinal polypeptide-like immunoreactivity E) Loss of neurons in the myenteric plexus

E) Loss of neurons in the myenteric plexus

A 16-year-old boy is brought to the physician by his father because he has no signs of puberty. Sexual development is Tanner stage 2. Physical examination shows a circumcised penis_ The testes are soft and small with a volume of 5 mL (N=20-30). The prostate is firm, nontender, and of appropriate size. There are no lesions or discharge. His serum testosterone concentration is in the prepubertal range. Measurement of which of the following hormones in serum is most likely to determine whether the decreased serum testosterone concentration is a result of a lack of pituitary hormone stimulus in this patient? A) Adrenocorticotropic hormone B) Estrogen C) Follicle-stimulating hormone D) Inhibin E) Luteinizing hormone

E) Luteinizing hormone

A 32-year-old woman cornea to the physician because of a 1-month history of progressive shortness of breath and anxiety. She says that her symptoms become much more pronounced as the day progresses. She is 170 cm (5 ft 7 in) tall and weighs 70 kg (155 lb): BMI is 24 kg/m2. Breath sounds are normal on auscultation of the chest, but the patient has difficulty taking a long, deep breath. Arterial blood gas analysis on room air shows: pH 7.33, Pco2 70 mm Hg, P02 65 mm Hg Pulmonary function testing shows a decreased vital capacity, tidal volume. and expiratory reserve volume. Her residual volume is within the reference range. Which of the following is the most likely underlying cause of this patient's condition? A) α-Antitrypsin deficiency B) Central nervous system neoplasm C) Chronic bronchitis D) Chronic opiate use E) Myasthenia gravis F) Obesity

E) Myasthenia gravis

A 24-year-old woman conies to the physician because of a 1-day history of pain in her left leg while walking. Her temperature is 37°C (98.6°F), pulse is 84/min, and blood pressure is 135/78 mm Hg, Examination of the left lower extremity shows only a markedly decreased pulse in the left lower extremity. An intermittent faint diastolic head murmur is heard at the apex. Angiography of the left lower extremity shows complete obstruction of the distal left femoral artery by what appears to be a thrombus. The occlusion is removed using a catheter, Microscopic examination of the removed specimen shows scattered mesenchymal cells in an abundant extracellular matrix, Echocardiography confirms a diagnosis of cardiac tumor. This patient most likely has which of the following types of cardiac tumors? A) Fibroelastoma B) Fibroma C) Hamartoma D) Hemangioma E) Myxoma

E) Myxoma

A 75-year-old woman with osteopenia comes to the physician for a follow-up examination. Current medications are occasional acetaminophen, and supplemental calcium and vitamin D. She does not smoke cigarettes or drink alcohol, She follows a high-protein diet and walks 3 miles daily. She is 157 cm (5ft 2 in) tall and weighs 59 kg (130 lb); BMI is 24 kg/m2. Her weight has not changed during the past 10 years. Her pulse is 68/min, and blood pressure is 124/72 mm Hg. Physical examination shows no abnormalities, Laboratory studies from today and 10 years ago show: Today 10 Years Ago Serum creatinine: 1.0 mg/dL 1.0 mg/dL Urine Creatinine excretion: 1000 mg/24 h 1200 mg/24 h Creatinine clearance: 69.4 mL/min 83 mL/min Which of the following is the most likely cause of this patient's decreased creatinine clearance rate? A) High-protein diet B) Over-supplementation with vitamin D C) Polycystic kidney disease D) Renovascular disease E) Normal aging

E) Normal aging

16. An 8-year-old girl is brought to the emergency department by her parents 15 minutes after she became unconscious. They had been traveling in their truck overnight and had made a bed for their daughter in the truck bed, which was enclosed in a cabin, to allow her to sleep during the journey. When they arrived home, they had difficulty arousing her. When she did awaken, she said that she had a severe headache and nausea She then began to vomit during ambulance transport to the hospital. Her pulse is 150/min. Physical examination shows cherry red skin around the mouth. The most appropriate immediate therapy should include which of the following? A) Amyl nitrite B) Dextrose C) Fomepizole D) Naloxone E) Oxygen

E) Oxygen

28. A 14-year-old boy has had tremors of the extremities and greenish rings at the margin of the cornea near the limbus for 1 year. Laboratory studies show a plasma ceruloplasmin concentration below the 95% confidence limits of normal and a 12-fold increase in serum ALT activity. Which of the following treatments is most likely to be effective? A) Calcium supplementation B) Multivitamins C) Oral administration of calcium disodium edetate (EDTA) D) Oral administration of ferrous salts E) Periodic intramuscular injections of penicillamine

E) Periodic intramuscular injections of penicillamine

49. A 22-year-old basketball player who uses cocaine recreationally suddenly develops severe shortness of breath after a routine scrimmage. Physical examination shows respirations of 25/min. The left lung field is hyperresonant and breath sounds are diminished. Which of the following is the most likely diagnosis? A) Acute respiratory distress syndrome B) Cor pulmonale C) Flash pulmonary edema D) Pleural effusion E) Pneumothorax

E) Pneumothorax

A 9-year-old boy is brought to the physician by his mother because of a 1-year history of cough productive of mucoid sputum, wheezing, and shortness of breath with exertion. He has a history of recurrent upper respiratory tract and sinus infections since birth, He is at the 25th percentile for height and weight. The mother says that his younger sibling is beginning to develop similar problems. Physical examination shows mild clubbing of the fingers. Laboratory studies show markedly increased sweat chloride and sodium concentrations. A defect of which of the following in this patient's bronchial epithelium is most likely causing these symptoms? A) Adrenoreceptors B) Membrane receptors C) Nuclear receptors D) Protein regulation E) Protein structure

E) Protein structure

A 23-year-old woman is treated with acetazolamide to prevent acute altitude sickness 3 days before going on a mountain-climbing expedition. Acetazolamide acts predominately in which of the following areas of the kidney in this patient? A) Ascending loop of Henle B) Collecting ducts C) Descending loop of Henle D) Distal tubule E) Proximal tubule

E) Proximal tubule

During an experimental study of oxygen consumption in the kidney, experimental animals are ventilated with 100% nitrogen. Cells from which of the following areas of the kidney are most likely to show the first signs of anoxic injury? A) Bowman capsule B) Distal convoluted tubule C) Efferent arteriole D) Glomerulus E) Proximal tubule

E) Proximal tubule

A 34 year old man is brought to the emergency department semiconscious and combative. In addition to sedation, a short-acting neuromuscular blocking agent is administered for intubation to prevent aspiration. Within a few seconds after administration of the drug , he has transient muscle fasciculations in his face; he develops generalized paralysis within 1 minute. Forty-five minutes after completion of the procedure, he is still paralyzed. A genetic abnormality of which of the following enzymes is the most likely cause of his unusually slow recovery from paralysis? A) Angiotensin-converting enzyme B) Choline O-acetyltransferase C) Monoamine oxidase D) Phenylethanolamine N-methyltransferase E) Pseudocholinesterase F) Tyrosine hydroxylase

E) Pseudocholinesterase

A 23-year-old man comes to the physician because of difficulty maintaining an erection. He was involved in an automobile collision that damaged his thoracic spinal cord, affecting motor and sensory function of the lower trunk and extremities. He is only able to obtain an erection through physical contact with his penis by his wife, and the erection does not last a long time. The best explanation for this type of erection is an intact spinal reflex arc between the sacral parasympathetic nerves and which of the following? A) Genitofemoral nerves B) Ilioinguinal nerves C) Lumbosacral trunks D) Obturator nerves E) Pudendal nerves

E) Pudendal nerves

41. A 70-year-old man is admitted to the hospital for evaluation of the recent loss of mental function. He has a history of weight loss. He is not taking any drugs. Vital signs are normal, and he is not dehydrated. Aside from mild disorientation, findings on neurologic examination are unremarkable. He has mild anemia. Laboratory studies show: Serum: Na+ 110 mEq/L, CL- 85 mEq/L, K+ 4.4 mEq/L, Urea nitrogen (BUN) 15 mg/dL, Creatinine 0.9 mg/dL, Plasma osmolality 250 mOsmol/kg, Urine osmolality 750 mOsmol/kg Which of the following is the most likely diagnosis? A) Adrenal carcinoma B) Chronic interstitial nephritis C) Diabetes mellitus D) Pneumococcal meningitis E) Pulmonary neoplasm

E) Pulmonary neoplasm

A 60-year-old man comes to the physician for a routine health maintenance examination, He has had normal blood pressure measurements. His blood pressure today is 170/95 mm Hg, Physical examination shows no other abnormalities, Serum studies show hypokalemia and metabolic alkalosis. Plasma renin activity and serum aldosterone concentrations are increased. Following the administration of captopril, there is a marked increase in plasma renin activity, Which of the following is the most likely cause of the findings in this patient? A) Aldosterone-secreting adrenal tumor B) Chronic glomerulonephritis C) Cushing syndrome D) Essential hypertension E) Renal artery stenosis

E) Renal artery stenosis

During an experiment, a cuff occluder is placed on the left renal artery of a normal rat and inflated to cause a 90% decrease in the cross-sectional area of the artery. A catheter is placed in the left and right renal veins to sample the effluent. The concentration of which of the following substances will most likely be higher in the effluent from the left renal vein than that from the right renal vein? A) ADH (vasopressin) B) Aldosterone C) Endothelin D) Epinephrine E) Renin

E) Renin

6. A newborn has cyanosis, tachypnea, and retractions of the muscles of the chest wall during inspiration. She weighs 2500 g (5 lb 8 oz). Arterial blood values while she breathes room air are: pH 7.04 Pco2 65 mm Hg Po2 35 mm Hg HCO3- 15 mEq/L Which of the following best describes her acid-base balance? A) Metabolic acidosis, uncompensated B) Metabolic alkalosis, uncompensated C) Respiratory acidosis, uncompensated C) Respiratory alkalosis, uncompensated E) Respiratory acidosis and metabolic acidosis F) Respiratory acidosis and metabolic alkalosis

E) Respiratory acidosis and metabolic acidosis

An 18-year-old woman receives general anesthesia for extraction of wisdom teeth. After 5 minutes, she has generalized hypertonicity of skeletal muscles and increased body temperature. The most appropriate treatment is a drug that decreases which of the following? A) Lactate production B) Motor axon acetylcholine release C) Nicotinic acetylcholine activation D) Prostaglandin synthesis E) Sarcoplasmic Ca 2+ release

E) Sarcoplasmic Ca 2+ release

A newborn has male genital ducts but female external genitalia. Cytogenetic analysis shows a 46,XY karyotype, and genetic testing shows a mutation of the gene encoding 5α-reductase In the absence of this mutation, the apparent labia majora of this newborn would have developed into which of the following structures? A) Glans penis B) Penile shaft C) Penile urethra D) Prostate utricle E) Scrotum

E) Scrotum

A 30-year-old woman comes to the physician because of a 6-month history of intermittent cutaneous flushing and diarrhea, An x-ray of the chest shows a solitary pulmonary nodule in the left lower lobe of the lung. Increased synthesis and release of which of the following compounds is most likely responsible for her symptoms? A) ACTH B) Dopamine C) Epinephrine D) Norepinephrine E) Serotonin

E) Serotonin

40. A 58-year-old African American man with congestive heart failure comes to the physician for a follow-up examination. Cardiac examination shows an S3. Echocardiography shows a left ventricular ejection fraction of 30%. Treatment is initiated with hydralazine and isosorbide dinitrate, Which of the following medications is contraindicated in this patient? A) Amiodarone B) Diltiazem C) Lisinopril D) Metoprolol E) Sildenafil

E) Sildenafil

A 24-year-old man comes to the physician with his 25-year-old wife because they have been unable to conceive for 3 years. The wife has been evaluated for infertility, and the test results were normal. Analysis of the man's semen shows normal numbers of living sperm but they are immotile. In addition to infertility, this man is most likely to have which of the following associated conditions? A) Cholelithiasis B) Coronary artery disease C) Fat malabsorption D) Glomerulonephritis E) Sinusitis

E) Sinusitis

A 50-year-old man comes to the physician because of a 2-week history of progressive shortness of breath while climbing stairs to his office every morning. He reports no other problems, but he is concerned because his father had a major myocardial infarction at the age of 52 years, His pulse is 110/min and regular, respirations are 16/min, and blood pressure is 135/95 mm Hg. The lungs are clear to auscultation. Cardiac examination shows normal heart sounds with a physiologic split of S2. Stress echocardiography shows hypokinesis of the posterior left ventricle with increasing activity levels. Which of the following is the most likely cause of the posterior left ventricular findings in this patient? A) Disruption of the sympathetic nerves to the left ventricle B) Extravascular compression of the coronary arteries C) Increased left ventricular end-diastolic pressure D) Increased myocardial oxygen consumption E) Stenosis of the right coronary artery

E) Stenosis of the right coronary artery

A 39-year-old woman with type 1 diabetes mellitus comes to the physician because of episodes of fainting during the past 3 weeks. The patient's insulin monitoring has shown that the episodes occur when she becomes hypoglycemic, In the past, her hypoglycemic episodes were characterized by increased sweating, heart rate, and shaking of her hands, but now she loses consciousness without warning, Current medications are insulin, lisinopril, and simvastatin. Her pulse is 72/min, and blood pressure is 128/70 mm Hg. Physical examination shows no abnormalities, It is most appropriate for this patient to receive an injection of a drug with which of the following mechanisms of action during these episodes? A) Decreased intestinal absorption of glucose B) Enhanced metabolism of insulin C) Promotion of peripheral insulin uptake D) Promotion of skeletal muscle glucose release E) Stimulation of hepatic glucose production

E) Stimulation of hepatic glucose production

46. Which of the following best explains the induction of galactorrhea by metoclopramide? A) Increased affinity of mammary gland prolactin receptors for prolactin B) Stimulation of release of estrogen from developing ovarian follicles C) Stimulation of release of growth hormone from the adenohypophysis D) Stimulation of release of oxytocin from the neurohypophysis E) Stimulation of release of prolactin from the adenohypophysis

E) Stimulation of release of prolactin from the adenohypophysis

A 27-year-old woman is admitted to the hospital because of a 12-hour history of fever and abdominal pain. She has a history of recurrent urinary tract infections. Her temperature is 39°C (102.2°F). Abdominal examination shows tenderness of the right flank. Abdominal x-rays show bilateral staghorn renal calculi, Urinalysis shows a pH of 8, many RBCs, WBCs, and bacteria, The calculi are most likely composed of which of the following in this patient? A) Calcium oxalate B) Calcium phosphate C) Cystine D) Hemoglobin E) Struvite F) Uric acid

E) Struvite

A 28-year-old man is undergoing evaluation for infertility. He is a weight lifter and takes anabolic steroids. Which of the following mechanisms best explains his infertility? A) Impaired liver metabolism B) Increased conversion of androgen to estrogen C) Increased conversion to dihydrotestosterone D) Increased occupancy of testosterone receptors in the testes E) Suppression of gonadotropins

E) Suppression of gonadotropins

A 44-year-old man comes to the physician because of a 2-month history of abdominal pain and diarrhea. The pain is temporarily relieved after eating and by the use of antacids. Physical examination shows epigastric tenderness. Laboratory studies show a serum gastrin concentration of 500 pg/mL (N=50-100) and gastric acid secretion of 80 mEq/h (N=6-40). Which of the following is the most definitive treatment at this time to decrease this patient's risk for complications? A) Low-protein diet B) Oral antibiotic therapy C) Oral antihistamine therapy D) Sectioning the vagus nerve to the stomach E) Surgical removal of the suspected tumor

E) Surgical removal of the suspected tumor

A 32-year-old man comes to the physician because of a 3-month history of swelling and tenderness of both breasts, He has been receiving thyroid hormone and corticosteroid replacement therapy since undergoing surgical removal of a pituitary adenoma 2 years ago. He began human chorionic gonadotropin (hCG) injections 4 months ago. Which of the following is the most likely binding site and action of hCG that is causing the development of gynecomastia in this patient? Tissue Effect A) Adrenal gland 17-hydroxyprogesterone synthesis B) Breast direct stimulation C) Liver testosterone metabolism D) Pituitary gland prolactin secretion E) Testicle estradiol production

E) Testicle estradiol production

A newborn is delivered at term to a 40-year-old primigravid woman. Chorionic villus sampling at 15 weeks' gestation was done and results of chromosomal analysis showed a male fetus. Physical examination shows female external genitalia and absent internal genitalia. Further tests are most likely to determine that this child has a mutation preventing responses to which hormone? A) Estrogen B) Follicle-stimulating hormone C) Luteinizing hormone D) Mullerian-inhibiting hormone E) Testosterone

E) Testosterone

A 16-year-old girl with cystic fibrosis is brought to the physician because of a 3-week history of generalized weakness, numbness and tingling of her arms and legs, and difficulty walking. She has not adhered to her medication regimen during the past 6 months. She appears alert and oriented. Her vital signs are within normal limits, Physical examination shows bilateral weakness and decreased deep tendon reflexes in the upper and lower extremities. She walks with an ataxic gait The most likely cause of these findings is a deficiency of which of the following? A) Folic acid B) Vitamin A C) Vitamin B6 (pyridoxine) D) Vitamin D E) Vitamin E

E) Vitamin E

27. A 54-year-old woman comes to the physician because of a 2-week history of heavy vaginal bleeding. Menopause occurred 5 years ago, She underwent a mastectomy 3 years ago for estrogen receptor-positive breast carcinoma with axillary lymph node involvement. She has been taking tamoxifen, a drug that blocks estrogen receptors in breast tissue, for the past 3 years. Biopsy of endometrial tissue shows hyperplasia. Which of the following best explains the development of endometrial hyperplasia in this patient? A) Endometrial tissues become hyperplastic in the absence of estrogen B) The patient has been noncompliant in taking the estrogen receptor antagonist C) The patient's breast cancer has metastasized to the patient's uterus D) The patient's breast cancer is associated with an estrogen-secreting ovarian cancer E) The patient's medication has an agonist effect on endometrial estrogen receptors F) The patient's serum estrogen concentrations are increased due to ectopic production of human chorionic gonadotropin

E) The patient's medication has an agonist effect on endometrial estrogen receptors

A newborn female is diagnosed with hypothyroidism 2 days after birth. Ultrasonography of the neck shows the absence of any thyroid gland tissue. Physical examination shows not abnormalities, including normal root, suck grasp, and Moro reflexes. Maternal to fetal transfer of which of the following best explains the normal development in this newborn? A) Iodine B) Thyroglobulin C) Thyroid-stimulating hormone D) Thyrotropin-releasing hormone E) Thyroxine (T4)

E) Thyroxine (T4)

15. The action potential causes current to flow deep within the skeletal muscle fiber by which of the following? A) Electrogenic Na+—K+ pump activity at the sarcoplasmic reticulum B) Electrotonic spread of the potential from one myofibril to the next C) Release of sodium from the sarcoplasmic reticulum D) Synchronous contraction of the myofibrils E) Transmission along T tubules

E) Transmission along T tubules

27. A 24-year-old woman participates in a study of renal function. Her renal oxygen consumption is 20 mL/min. Which of the following processes uses the most oxygen consumption in the kidneys of this woman? A) Acid-base homeostasis B) Glomerular filtration C) Metabolism of tubular toxins D) Production of renal hormones E) Tubular reabsorption

E) Tubular reabsorption

48. An autopsy is performed on a previously healthy 20-year-old woman who died immediately following a motor vehicle collision 8 hours ago. Microscopic examination of the lungs is most likely to show that the greatest amount of alveolar endothelial surface area is covered by which of the following cells? A) Brush cells B) Fibroblasts C) Lymphatic capillary cells D) Macrophages E) Type I pneumocytes

E) Type I pneumocytes

A 49-year-old man is brought to the emergency department 30 minutes after fainting in the street, He regained consciousness 1 minute after fainting. He says that he has had watery diarrhea during the past 5 days, which has not improved with fasting. He has not changed his diet or travelled overseas recently, His blood pressure is 90/60 mm Hg. Physical examination shows a flushed face and dehydration, Serum studies show a potassium concentration of 2 mEq/L and glucose concentration of 150 mg/dL. He is admitted to the hospital, and intravenous fluid replacement is started, Over the next day, he passes a stool with a volume of 3.5 L. Which of the following is the most likely cause of this patient's diarrhea? A) Bacterial overgrowth B) Cholera C) Inactivation of lipase D) Lactose intolerance E) Vasoactive intestinal polypeptide secretion

E) Vasoactive intestinal polypeptide secretion

37. A 54-year-old woman with hypertension and bilateral renal artery stenosis starts taking a nonsteroidal anti-inflammatory drug for back pain. Over the next week, her serum creatinine concentration increases from 1.0 to 5.0mg/dL. The most likely cause of this finding is the drug's ability to inhibit which of the following? A) Inflammation in the glomerular capillaries B) Inflammation in the renal interstitium C) Vasoconstricting prostaglandins at the afferent arteriole D) Vasoconstricting prostaglandins at the efferent arteriole E) Vasodilating prostaglandins at the afferent arteriole F) Vasodilating prostaglandins at the efferent arteriole

E) Vasodilating prostaglandins at the afferent arteriole

44. A 3-month-old boy is brought to the physician by his mother because of a 1-week history of difficulty breast-feeding. The mother reports that her son frequently stops breast-feeding and seems to be gasping for breath. The amount of time he spends feeding seems less than that of her other children. He is at the 5th percentile for length and 5th percentile for weight. The lungs are clear to auscultation. A grade 3/6 holosystolic murmur is heard. Subsequent cardiac test results show normal oxygen (O2) tension in the right atrium, increased O2 tension in the right ventricle, and normal systemic O2 tension. Which of the following is the most likely diagnosis? A) Atrial septal defect B) Coarctation of the aorta C) Tetralogy of Fallot D) Transposition of the great arteries E) Ventricular septal defect

E) Ventricular septal defect

A 2-month-old boy is brought to the physician for a well-child examination, The mother has no concerns about his growth or breast-feeding habits and says that he has begun to smile. He is at the 25th percentile for length and 30th percentile for weight, Cardiac examination shows a blowing holosystolic murmur heard best over the lower left sternal border. Which of the following is the most likely cause of the cardiac findings in this patient? A) Coarctation of the aorta B) Functional murmur C) Patent ductus arteriosus D) Patent foramen ovale E) Ventricular septal defect

E) Ventricular septal defect

39. A 26-year-old woman has lost 95 kg (209 lb) since undergoing a jejunoileal bypass for treatment of morbid obesity Fecal fat excretion is excessive. This patient most likely has a deficiency in which of the following nutrients? A) Iron B) Magnesium C) Niacin D) Protein E) Vitamin A F) Vitamin B1 (thiamine)

E) Vitamin A

A 14-year-old boy has had fatigue, intermittent right lower quadrant abdominal pain, diarrhea, and a 5-kg (11-lb) weight loss over the past 6 months, A diagnosis of inflammatory bowel disease limited to the terminal ileum is made. Absorption of which of the following is most likely to be impaired in this patient? A) Folic acid B) Iron C) Vitamin B1 (thiamine) D) Vitamin B2 (riboflavin) E) Vitamin B12 (cobalamin)

E) Vitamin B12 (cobalamin)

A 55-year-old man comes to the physician because of a 3-day history of fever, chills, nausea, vomiting, and diarrhea. He says he has been unable to eat or drink anything without vomiting and has had little urine production during the past 12 hours. His temperature is 37.8°C (100°F). His pulse is 92/min, and blood pressure is 110/70 mm Hg while supine: pulse is 110/min, and blood pressure is 80/60 mm Hg while standing. Physical examination shows dry mucous membranes and a soft abdomen with mild, diffuse tenderness. Laboratory studies show: Leukocyte count 7200/mm3, Serum: Na+ 146 mEq/L, Urea nitrogen 50 mg/dL, Creatinine 2.2 mg/dL, Uric acid 10.2 mg/dL. Urine: Specific gravity 1.030, RBC O/hpf, Protein negative, Na+ 10 mEq/L, Creatinine 19 mg/dL. Which of the following is the most likely cause of the laboratory findings in this patient? A) Acute tubular necrosis B) Bladder outlet obstruction C) Interstitial nephritis D) Membranous glomerulonephritis E) Volume depletion

E) Volume depletion

A 40-year-old woman with a 6-month history of episodic sinusitis comes to the physician because of a 2-week history of intermittent headaches, fatigue, and generalized joint pain. She has had a worsening cough that has recently become productive of blood-tinged sputum. This is her fourth visit to the physician in the past 4 months. Therapy with antibiotics, decongestants, and nasal corticosteroids has not resulted in improvement in her sinus-related symptoms. Physical examination shows erythema of the nasal mucosa and two small ulcerations, Auscultation of the lungs shows scattered crackles. An x-ray of the chest shows patchy opacities bilaterally. Laboratory studies show: Hemoglobin 13 g/dL, Erythrocyte sedimentation rate 70 mm/h, Leukocyte count 10,500/mm3, Serum antineutrophil cytoplasmic antibody increased. Which of the following is the most likely diagnosis? A) Allergic bronchopulmonary aspergillosis B) Churg-Strauss syndrome C) Sarcoidosis D) Tuberculosis E) Wegener granulomatosis

E) Wegener granulomatosis

46. A 73-year-old man comes to the physician because of a 4-month history of localized mid-back pain. He has hypertension and type 2 diabetes mellitus. He is in mild discomfort but is able to walk without problems, There is pain with percussion over the lower thoracic spine. Neurologic examination shows no focal findings. X-rays of the spine show vertebral cortical thickening of the end-plates, creating "picture frame" vertebrae, Which of the following sets of serum studies is most likely in this patient? Calcium Phosphorus Alkaline Phosphatase Parathyroid Hormone A) UP no change no change no change B) UP DOWN no change UP C) UP no change no change DOWN D) no change no change UP UP E) no change no change UP no change

E) no change no change UP no change

During an experiment, an isolated skeletal muscle is placed in a bath with a high concentration of glucose for 2 hours. The muscle is then contracted with electrical simulation. An intracellular decrease in which of the following most likely indicates the onset of muscle fatigue? A) ADP B) Inorganic phosphate C) Inosine monophosphate D) Lactate E) pH

E) pH

48. Adhesion of platelets to the subendothelial surface requires the interaction of platelet membrane glycoprotein with which of the following? A) Fibrinogen B) Spectrin C) Thrombomodulin D) Thromboplastin E) von Willebrand factor

E) von Willebrand factor

A 45-year-old man comes to the physician because of an enlarging face, shoulders, and trunk and thinning of his arms and legs. Physical examination shows a round, plethoric face, fat pad over the upper thoracic spine, and purple striae on the abdomen. Serum studies show an undetectable adrenocorticotropic hormone (ACTH) concentration and an increased cortisol concentration, Administration of low-dose dexamethasone would most likely result in which of the following sets of serum findings? ACTH Cortlsol A)UP. UP B)UP no change C)UP. DOWN D)No change UP E)No change no change F)No change DOWN

E)No change no change

A 14-year-old girl with type 1 diabetes mellitus is brought to the emergency department because of a 4-hour history of lethargy, confusion, and disorientation. She has vomited twice during this period, and she says she is thirsty and that her stomach hurts. The symptoms developed gradually overnight after she did not take her usual insulin dose during a sleepover at her friend's house. Her pulse is 110/min; respirations are 24/min, deep, and rapid; and blood pressure is 95/75 mm Hg. Laboratory studies show a serum glucose concentration of 450 mg/dL and arterial pH of 7.15. Arterial blood gas analysis on room air most likely includes which of the following additional findings? Arterial Pco2 Arterial HCO3- Anion Gap A) UP normal normal B) UP DOWN UP C) UP DOWN normal D) DOWN UP UP E) DOWN normal normal F) DOWN DOWN UP G) DOWN DOWN normal

F) DOWN DOWN UP

A 23-year-old woman. gravida 1. para 1, is brought to the physician because she has not fell well since the delivery of her son 2 weeks ago; she has had ongoing fatigue, inability to breast-feed, and episodic light-headedness when in the upright position during this period, Pregnancy was complicated by preeclampsia and resulted in an urgent cesarean delivery. Immediately after delivery, she required blood transfusions because of bleeding and transient hypotension. She was discharged from the hospital after she had a stable hemoglobin concentration of 10.8 g/dl. She appears lethargic. While she is sitting, her pulse is 80/min, and blood pressure is 100/50 mm Hg: while she is standing, pulse is 85/min, and blood pressure is 86/44 mm Hg, Physical examination shows no other abnormalities. Her hemoglobin concentration is 11.6 g/dL, and hematocrit is 35%. Which of the following sets of serum hormone concentrations is most likely in this patient? Prolactin Adrenocorticotropic Hormone Thyroid-Stimulating Hormone Aldosterone A) UP UP normal normal B) UP normal normal normal C) UP DOWN UP. DOWN D) DOWN normal normal normal E) DOWN. DOWN. Normal normal F) DOWN. DOWN. DOWN. UP

F) DOWN. DOWN. DOWN. UP

A 29-year-old woman comes to the physician because of a 5-week history of fatigue and a 4-day history of heart palpitations and anxiety, She has primary hypothyroidism treated with triiodothyronine, She says that she has doubled the dose of the medication over the past week because of fatigue. Her pulse is 112/min, and blood pressure is 126/70 mm Hg. Physical examination shows a fine motor tremor of the hands. Deep tendon reflexes are brisk. Thyroid function testing is most likely to show which of the following sets of serum concentrations? Thyroid-stimulating Hormone; Free Thyroxine; Free Triiodothyronine A) Increased; increased; increased B) Increased; increased; decreased C) Increased; decreased; increased D) Decreased; increased; increased E) Decreased; increased; decreased F) Decreased; decreased; increased

F) Decreased; decreased; increased

A 64-year-old man is evaluated for cough, dyspnea, and chest pain. He is afebrile, An x-ray of the chest shows an abnormal density in the hilar region, Lung tissue biopsy shows round cells with little cytoplasm that are about twice the size of lymphocytes, These cells are arranged in infiltrating sheets that have neither glandular nor squamous organization. Which of the following abnormalities in serum is most likely in this patient? A) Hyperkalemia B) Hyperlipidenia C) Hypermagnesemia D) Hypoalbuminemia E) Hypoglycemia F) Hyponatremia

F) Hyponatremia

27. A 29-year-old woman comes to the physician because of inability to conceive children. She has had 10 menstrual periods in the past year, but her menstrual pattern is very erratic and unpredictable. She has been advised that she may not be ovulating, She had some right pelvic pain 4 days ago. and she suspects that this pain is an indication that she has ovulated. Which of the following changes in serum hormone concentration is the best evidence that ovulation has occurred? A) Decrease in estradiol B) Decrease in follicle-stimulating hormone C) Decrease in progesterone D) Increase in estradiol E) Increase in follicle-stimulating hormone F) Increase in progesterone

F) Increase in progesterone

43. A 3-year-old girl is brought to the emergency department by her mother 1 hour after she was found with a half-empty bottle of her grandmother's diabetes medication. The mother tells the physician that the child consumed approximately 25 Metformin tablets. Physical examination shows no abnormalities. This patient is at greatest risk for which of the following serum abnormalities? A) Decreased calcium concentration B) Decreased glucose concentration C) Decreased sodium concentration D) Increased AST and ALT activities E) Increased creatinine concentration F) Increased lactic acid concentration

F) Increased lactic acid concentration

A 47-year-old man is admitted to the hospital for treatment of a myocardial infarction_ On admission, pulse oximetry on 30% oxygen shows an oxygen saturation greater than or equal to 95%, Three hours later, the patient develops shortness of breath. Pulse oximetry now shows an oxygen saturation of 90%. Crackles are heard at the lung bases, and a grade 2/6 systolic murmur is heard, Arterial blood gas analysis on 30% oxygen shows: pH 7.41, Pco2 36 mm Hg, Po2 60 mm Hg Which of the following is the most likely cause of this patient's hypoxemia? A) Decreased alveolar ventilation B) Decreased erythrocyte transit time in pulmonary capillaries C) Decreased lymphatic drainage D) Increased permeability of pulmonary capillaries E) Increased plasma colloid osmotic pressure F) Increased pulmonary capillary pressure

F) Increased pulmonary capillary pressure

A 66-year-old woman with ovarian cancer comes to the emergency department because of the inability to urinate for 2 days and bilateral flank pain for 8 hours. She has not had suprapubic pain. Vital signs are normal. Physical examination shows bilateral costovertebral angle tenderness. Insertion of a Foley catheter yields no urine. Renal ultrasonography shows bilateral hydronephrosis. Which of the following is most likely present in this patients kidney? A) Decreased glomerular oncotic pressure B) Decreased interstitial hydrostatic pressure C) Decreased tubular hydrostatic pressure D) Increased glomerular oncotic pressure E) Increased interstitial oncotic pressure F) Increased tubular hydrostatic pressure

F) Increased tubular hydrostatic pressure

A 22-year-old woman is admitted to the hospital because of a 10-day history of polydipsia and polyuria. She says that the urge to urinate often awakens her at night. She has been taking lithium carbonate for 2 years for bipolar disorder; her dosage was increased 6 months ago because of recurrent severe manic episodes. Her vital signs are within normal limits. Physical examination shows no abnormalities. Over the next 24 hours, urine excretion totals 6.5 L. Laboratory studies at this time show a serum sodium concentration of 148 mEq/L, serum osmolality of 315 mOsmol/kg, and urine osmolality of 75 mOsmol/kg. After administration of desmopressin, urine output and osmolality do not change. Which of the following findings in the nephron best describes the tubular osmolality, compared with serum, in this patient? Proximal Tubule Juxtaglomerular Apparatus Medullary Collecting Duct A) Hypertonic hypertonic hypertonic B) Hypertonic hypertonic hypotonic C) Hypertonic hypotonic hypotonic D) Isotonic isotonic isotonic E) Isotonic hypotonic hypertonic F) Isotonic hypotonic hypotonic G) Hypotonic hypertonic hypertonic H) Hypotonic hypotonic hypertonic I) Hypotonic hypotonic hypotonic

F) Isotonic hypotonic hypotonic

A 52-year-old man is brought to the physician 3 days after the sudden onset of blindness of the left eye. He does not have any eye pain. Funduscopy of the left eye shows a pale, opaque fundus and a bright red fovea centralis, Visual field testing shows a dense scotoma of the entire visual field of the left eye; testing of the right eye shows no abnormalities, At a follow-up examination 6 months later, the patient remains blind in the left eye. If the left eye is illuminated, which of the following reactions is most likely in the right pupil of this patient? A) Constriction because the left optic tract is binocular B) Constriction because projections to the Edinger-Westphal nucleus are bilateral C) Dilation because the posterior commissure is intact D) Dilation because the right superior cervical ganglion is intact E) No constriction because the left ciliary nerve has been permanently damaged F) No constriction because the retinal ganglion cells in the left eye have been destroyed

F) No constriction because the retinal ganglion cells in the left eye have been destroyed

23. A healthy 21-year-old man dies suddenly of cardiac arrest after snorting cocaine. Cocaine-induced inhibition of which of the following is the most likely explanation for the cardiac arrest? A) Acetylcholinesterase B) α2-Adrenergic receptors C) ß2-Adrenergic receptors D) Monoamine oxidase E) Muscarinic receptors F) Norepinephrine reuptake

F) Norepinephrine reuptake

An 18-year-old man comes to the physician because of a 1-year history of progressive headaches. An x-ray of the head shows a cystic tumor above the sella turcica. The serum concentration of which of the following hormones is most likely to be increased in this patient? A) ADH (vasopressin) B) Adrenocorticotropic C) Follicle-stimulating D) Growth E) Luteinizing F) Prolactin G) Thyroid-stimulating

F) Prolactin

31. A 6-week-old boy has a 2-week history of projectile vomiting after feeding. Several changes in formula do not change his symptoms_ Vomitus is free of bile. He is dehydrated. and stools are decreased. Serum electrolyte and pH findings show metabolic alkalosis. Which of the following is the most likely diagnosis? A) Achalasia B) Congenital megacolon (Hirschsprung disease) C) Meconium ileus D) Necrotizing enterocolitis E) Paralytic ileus F) Pyloric stenosis

F) Pyloric stenosis

49. A 30-year-old man is brought to the emergency department 30 minutes after being stung by several wasps. He is confused and has difficulty breathing. His temperature is 38°C (100.4°F), pulse is 122/min, respirations are 34/min, and blood pressure is 80/40 mm Hg. Physical examination shows dry skin and decreased capillary refill. There are multiple erythemalous, inflamed marks on the back and 1+ pitting edema of the ankles, In addition to the administration of 0.9% saline, the most appropriate next step in management is administration of which of the following? A) ACE inhibitor B) Angiotensin II receptor blocker C) Parasympathetic blocker D) Parasympathomimetic agent E) Sympathetic blocker F) Sympathomimetic agent G) Whole blood

F) Sympathomimetic agent

A 40-year-old man comes to the physician because of a 6-month history of difficulty maintaining an erection during sexual intercourse. He has consumed 1/2 L of bourbon daily for 15 years. His pulse is 88/min, and blood pressure is 130/80 mm Hg. Examination shows scleral icterus and spider angiomata over the trunk. The liver span is 5 to 6 cm in the midclavicular line. The spleen tip is palpated 5 to 6 cm below the left costal margin. Decreased serum concentrations of which of the following is the most likely cause of this patient's erectile dysfunction? A) Estradiol B) Estrone C) Follicle-stimulating hormone D) Human chorionic gonadotropin E) Luteinizing hormone F) Testosterone

F) Testosterone

34. An investigator is studying ß2-adrenoreceptors in female experimental animals. During the experiment, epinephrine is injected intramuscularly into each animal, and the effects on ß2-adrenoreceptors are then observed. Which of the following physiologic effects is most likely to be observed in these animals? A) Increased myocardial contractility B) Internal urethral sphincter contraction C) Lipolysis D) Pilomotor contraction E) Pupillary dilation F) Uterine relaxation

F) Uterine relaxation

A 49-year-old man with pancreatic cancer undergoes a Whipple procedure to resect the distal stomach, duodenum, and head of the pancreas. During the procedure, the cut end of the remaining stomach is joined to the side of the jejunum (gastrojejunostony). Which of the following nerve structures can be cut to decrease stomach acid output and decrease the likelihood of ulcers forming in the jejunum at the site of junction with the stomach? A) Celiac plexus B) Iliohypogastric nerves C) Phrenic nerves D) Subcostal nerves E) Sympathetic chain F) Vagal trunks

F) Vagal trunks

A 38-year-old man is admitted to the hospital after sustaining a gunshot wound to the abdomen. Broad-spectrum antibiotic therapy is initiated for complications from fecal contamination of the peritoneal cavity. He has been taking warfarin since receiving an artificial heart valve 6 years ago because of endocarditis secondary to intravenous drug use. During the next 6 weeks, the warfarin dosage required to maintain his prothrombin time progressively decreases. Which of the following best explains this finding? A) Decreased INR caused by drug-induced hepatotoxicity B) Decreased INR caused by hepatitis B infection C) Septic shock caused by an anaerobic commensal such as Bifidobacterium species D) Septic shock caused by a facultative anaerobe such as Escherichia coif E) Vitamin K deficiency caused by bacterial overgrowth in the small intestine F) Vitamin K deficiency caused by depletion of the normal gut flora

F) Vitamin K deficiency caused by depletion of the normal gut flora

A 33-year-old woman who is right-handed is brought to the physician because of a 3-day history of progressive weakness and numbness of her arms and legs. Neurologic examination shows proximal and distal weakness of the upper and lower extremities. There is areflexia. Sensation to vibration and joint position is decreased in the fingers and toes. Nerve conduction studies show a slow conduction velocity in the median, ulnar, peroneal, and tibial nerves. These electrophysiologic findings most likely indicate impaired function of which of the following ion channels? A) Neurotransmitter-gated Ca2+ channels B) Neurotransmitter-gated K+ channels C) Neurotransmitter-gated Na+ channels D) Voltage-gated Ca2÷ channels E) Voltage-gated K+ channels F) Voltage-gated Na+ channels

F) Voltage-gated Na+ channels

A 35-year-old man with a long-standing history of chronic atrophic gastritis comes to the physician because of increased lethargy and dizziness during the past 2 weeks. Physical examination shows pale skin and mucous membranes. There is no palpable adenopathy. The lungs are clear to auscultation. Heart sounds are normal. The liver edge is palpable. Test of the stool for occult blood is negative. Neurologic examination shows decreased vibratory sensation. Laboratory studies show a hematocrit of 27%, mean corpuscular volume of 112 µm3, and platelet count of 200,000/mm3. The absence of sufficient numbers of which of the following cell types best explains these findings? A) Chief B) Columnar absorptive C) Enteroendocrine D) Goblet E) Mucous F) Paneth G) Parietal H) Undifferentiated crypt

G) Parietal

A 26-year-old woman comes to the physician's office because of a 13.5-kg (30-lb) weight gain, irregular menstrual periods, constipation, and fatigue for the past 6 months. She also complains of increased difficulties with concentration and memory for the past month. Physical examination shows a blood pressure of 145/84 mm Hg. dry skin, and decreased muscle stretch reflexes at the ankles. Serum cholesterol concentration is 310 mg/dL, and serum potassium concentration is 4.0 mEq/L, Determination of which of the following serum concentrations is most likely to confirm the diagnosis? A) Cortisol B) Dehydroepiandrosterone C) Hemoglobin A1C D) Luteinizing hormone E) Prolactin F) Testosterone G) Thyroid-stimulating hormone

G) Thyroid-stimulating hormone

A 45-year-old man comes to the physician for a follow-up examination 2 weeks after beginning treatment with hydroohlorothiazide for hypertension. He says that he feels faint, light-headed, and dizzy when he gets out of bed and when he rises from a seated position too quickly. His pulse is 75/min, respirations are 12/min, and blood pressure is-130/85 mm Hg while supine. Physical examination shows no abnormalities. Which of the following sets of changes best characterizes changes in the cardiovascular system as this man goes from the supine to the standing position? Venous Return Carotid Sinus Baroreceptor Activity Cerebral Blood Flow A) UP UP UP B) UP UP DOWN C) UP DOWN UP D) UP DOWN DOWN E) DOWN UP UP F) DOWN UP DOWN G) DOWN DOWN UP H) DOWN DOWN DOWN

H) DOWN DOWN DOWN

A 2-year-old boy is brought to the physician because of increased thirst, urinary frequency, and failure to thrive. He appears frail. He is at the 5th percentile for length and 7th percentile for weight. His temperature is 38C (100.4F). Physical examination shows dehydration and decreased muscle tone. The diagnosis of Fanconi syndrome is made. Which of the following sets of changes in fractional reabsorption in the kidneys is most likely in this patient? Amino Acids; Glucose; Phosphate; HCO3- A) Increased; increased; increased; increased B) Increased; increased; decreased; decreased C) Increased; decreased; increased; increased D) Increased; decreased; decreased; decreased E) Decreased; increased; increased; increased F) Decreased; increased; decreased; decreased G) Decreased; decreased; increased; decreased H) Decreased; decreased; decreased; decreased

H) Decreased; decreased; decreased; decreased

A 35-year-old woman has congestive cardiomyopathy and pitting edema. Her serum urea nitrogen concentration is 25 mg/dL, and serum creatinine concentration is 1.8 mg/dL. Furosernide therapy is started. Five days later, laboratory studies show: Serum: Na+ 130 mEq/L, K+ 4.5 mEq/L, Cl- 90 mEq/L, HCO3- 30 mEq/L, Urea nitrogen 85 mg/dL, Creatinine 2.2 mg/dL, Albumin 3 g/dL Urine: Specific gravity 1.023, RBC 0/hpf, WBC 0-1/hpf, Sediment none The urinary fractional excretion of sodium is less than 1%. Which of the following is the most likely explanation for the observed decrease in renal function? A) Acute tubular necrosis F) Obstructive renal failure B) Diabetic nephropathy G) Osmotic diuresis C) Glomerulonephritis H) Prerenal azotemia D) Hepatorenal syndrome I) Renal artery stenosis E) Nephrogenic diabetes insipidus J) Syndrome of inappropriate secretion of ADH (vasopressin)

H) Prerenal azotemia

20. A 32-year-old man with an X-linked recessive disease has deafness, hematuria, and progressive renal failure. He most likely has an abnormality in which of the following proteins? A) Fibrillin B) Fibronectin C) Keratin D) Laminin E) Osteocalcin F) Thrombospondin G) Tropoelastin H) Type IV collagen

H) Type IV collagen

48. A 3-year-old girl is found to have a grade 4/6, loud, harsh, high-pitched holosystolic murmur that radiates over the precordium and a palpable thrill at the left sternal border. Which of the following defects is most likely in this patient? A) Aortic regurgitation F) Mitral stenosis B) Aortic stenosis D) Patent ductus arteriosus C) Atrial septal defect H) Pulmonic stenosis D) Coarctation of the aorta I) Tricuspid regurgitation E) Mitral regurgitation J) Ventricular septal defect

J) Ventricular septal defect

A 56 year old man is brought to the hospital because of lethargy for the past month. He is not taking any medications. He does not have postural hypotension or edema. Laboratory sudies show Na+ 125 mEq/L, Creatinine 1 mg/dL, Thyroxine (T4) 10 µg/dL, Cortisol (AM) 20 µg/dL, Plasma osmolality 275 mOsmol/kg, Urine osmolality 450 mOsmol/kg, Urin Na+ 30 mEq/L. These findings are most likely associated with which of the following neoplasms?

Small cell carcinoma of the lung


Related study sets

Chapter 33: Skin Integrity and Wound Care

View Set

Ok State - BIOL 1114 - Final Exam

View Set

Module 12 - Application Layer Services

View Set

Health Promotion in Older Adult Quiz 20

View Set

Finance: 12-16 + 19 Concept Questions

View Set

Business Analytics II Chapter 13

View Set

Culture (assessment project questions)

View Set